Исследование функции как найти наибольшее значение

На практике довольно часто приходится использовать производную для того, чтобы вычислить самое большое и самое маленькое значение функции. Мы выполняем это действие тогда, когда выясняем, как минимизировать издержки, увеличить прибыль, рассчитать оптимальную нагрузку на производство и др., то есть в тех случаях, когда нужно осуществить поиск и определить оптимальное значение какого-либо параметра или количество. Чтобы решить такие задачи верно, надо хорошо понимать, что такое наибольшее и наименьшее значение функции.

Обычно нами строится выражение этих значений в рамках некоторого интервала x, который может в свою очередь соответствовать всей области определения функции или ее части. Это может быть как отрезок [a; b], так и открытый интервал (a; b), (a; b], [a; b), бесконечный интервал (a; b), (a; b], [a; b) либо бесконечный промежуток -∞; a, (-∞; a], [a; +∞), (-∞; +∞).

В этом материале мы расскажем, как найти наибольшее и наименьшее значение явно заданной функции с одной  переменной y=f(x)y=f(x), чтобы вам не нужно было искать это самостоятельно онлайн.

Основные определения

Начнем, как всегда, с формулировки основных определений: какое значение называют максимальным и минимальным?.

Определение 1

Наибольшее значение функции y=f(x) на некотором промежутке x – это значение max y=f(x0)x∈X, которое при любом значении xx∈X, x≠x0 делает справедливым неравенство f(x)≤f(x0).

Определение 2

Минимальное значение функции y=f(x) на некотором промежутке x– это значение minx∈Xy=f(x0), которое при любом значении x∈X, x≠x0 делает справедливым неравенство f(Xf(x)≥f(x0).

Данные определения являются достаточно очевидными. Еще проще можно сказать так: наибольшее значение функции – это ее наибольшее число, которое она может принимать на известном интервале при абсциссе x0, а наименьшее – это самое маленькое принимаемое значение на том же интервале при x0.

Определение 3

Стационарными точками называются такие значения аргумента функции, при которых ее производная обращается в 0.

Зачем нам нужно знать, что такое стационарные точки?  Для ответа на этот вопрос надо вспомнить теорему Ферма. Из нее следует, что стационарная точка – это такая точка, в которой находится экстремум дифференцируемой функции (т.е. ее локальный минимум или максимум). Следовательно, функция будет принимать наименьшее или то, что больше всего, значение на некотором промежутке именно в одной из стационарных точек.

Еще  функция может принимать наибольшее или наименьшее значение в тех точках, в которых сама функция является определенной, а ее первой производной не существует.

Первый вопрос, который возникает при изучении этой темы: во всех ли случаях мы можем определить наибольшее или найти наименьшее значение функции на заданном отрезке? Нет, мы не можем этого сделать тогда, когда границы заданного промежутка будут совпадать с  границами области определения, или если мы имеем дело с интервалом, не имеющим конца. Бывает и так, что функция в заданном отрезке или на бесконечности будет принимать бесконечно малые или бесконечно большие значения (мало и много). В этих случаях определить или найти наибольшее и/или наименьшее значение не представляется возможным.

Более понятными эти моменты станут после изображения на графиках:

Наибольшее и наименьшее значение функции на отрезке

Наибольшее и наименьшее значение функции на отрезке

Первый рисунок показывает нам функцию, которая принимает наибольшее и наименьшее значения (max y и min y) в стационарных точках, расположенных на отрезке [-6;6].

Разберем подробно случай, указанный на втором графике. Изменим значение отрезка на [1;6] и получим, что наибольшее значение функции будет достигаться в точке с абсциссой в правой границе интервала, а наименьшее – в стационарной точке.

На третьем рисунке абсциссы точек представляют собой граничные точки отрезка [-3;2]. Они соответствуют наибольшему и наименьшему значению заданной функции.

Наибольшее и наименьшее значение функции на открытом интервале

Наибольшее и наименьшее значение функции на открытом интервале

Теперь посмотрим на четвертый рисунок. В нем функция принимает max y (наибольшее значение) и min y (наименьшее значение) в стационарных точках на открытом интервале (-6;6).

Если мы возьмем интервал [1;6), то можно сказать, что наименьшее значение функции на нем будет достигнуто в стационарной точке. Наибольшее значение нам будет неизвестно. Функция могла бы принять наибольшее значение при x, равном 6, если бы x=6 принадлежала интервалу. Именно этот случай нарисован на графике 5.

На графике 6 наименьшее значение данная функция приобретает в правой границе интервала (-3;2], а о наибольшем значении мы не можем сделать определенных выводов.

Наибольшее и наименьшее значение функции на бесконечности

Наибольшее и наименьшее значение функции на бесконечности

На рисунке 7 мы видим, что функция будет иметь max y в стационарной точке, имеющей абсциссу, равную 1. Наименьшего значения функция достигнет на границе интервала с правой стороны. На минус бесконечности значения функции будут асимптотически приближаться к y=3.

Если мы возьмем интервал x∈2; +∞, то увидим, что заданная функция не будет принимать на нем ни наименьшего, ни наибольшего значения. Если x стремится к 2, то значения функции будут стремиться к минус бесконечности, поскольку прямая x=2 – это вертикальная асимптота. Если же абсцисса стремится к плюс бесконечности, то значения функции будут асимптотически приближаться к y=3. Именно этот случай изображен на рисунке 8.

Как найти наибольшее и наименьшее значение непрерывной функции на заданном отрезке

Как найти наибольшее и наименьшее значение функции на отрезке?

В этом пункте мы приведем последовательность действий, которую нужно выполнить, чтобы найти наибольшее значение функции на некотором отрезке или как найти наименьшее значение функции.

  1. Для начала найдем область определения функции. Проверим, входит ли в нее заданный в условии отрезок.
  2. Теперь вычислим точки, содержащиеся в данном отрезке, в которых не существует первой производной. Чаще всего их можно встретить у функций, аргумент которых записан под знаком модуля, или у степенных функций, показатель которых является дробно рациональным числом.
  3. Далее выясним, какие стационарные точки попадут в заданный отрезок. Для этого надо вычислить производную функции, потом приравнять ее к 0 и решить получившееся в итоге уравнение, после чего выбрать подходящие корни. Если у нас не получится ни одной стационарной точки или они не будут попадать в заданный отрезок, то мы переходим к следующему шагу.
  4. Определим, какие значения будет принимать функция в заданных стационарных точках (если они есть), или в тех точках, в которых не существует первой производной (если они есть), либо же вычисляем значения для x=a и x=b.
  5. У нас получился ряд значений функции, из которых теперь нужно выбрать самое больше и самое маленькое. Это и будут наибольшее и наименьшее значения функции, которые нам нужно найти.

Посмотрим, как правильно применить этот алгоритм при решении задач.

Пример 1

Условие: задана функция y=x3+4×2. Определите ее наибольшее и наименьшее значение на отрезках [1;4] и [-4;-1].

Решение:

Начнем с нахождения области определения данной функции. В этом случае ей будет множество всех действительных чисел, кроме 0. Иными словами, D(y): x∈(-∞; 0)∪0; +∞. оба отрезка, заданных в условии, будут находиться внутри области определения.

Теперь вычисляем производную функции согласно правилу дифференцирования дроби:

y’=x3+4×2’=x3+4’·x2-x3+4·x2’x4==3×2·x2-(x3-4)·2xx4=x3-8×3

Мы узнали, что производная функции будет существовать во всех точках отрезков [1;4] и [-4;-1].

Теперь нам надо определить стационарные точки функции. Сделаем это с помощью уравнения x3-8×3=0. У него есть только один действительный корень, равный 2. Он будет стационарной точкой функции и попадет в первый отрезок [1;4].

Вычислим значения функции на концах первого отрезка и в данной точке, т.е. для x=1, x=2 и x=4:

y(1)=13+412=5y(2)=23+422=3y(4)=43+442=414

Мы получили, что наибольшее значение функции max yx∈[1; 4]=y(2)=3 будет достигнуто при x=1, а наименьшее min yx∈[1; 4]=y(2)=3 – при x=2.

Второй отрезок не включает в себя ни одной стационарной точки, поэтому нам надо вычислить значения функции только на концах заданного отрезка:

y(-1)=(-1)3+4(-1)2=3

Значит,  max yx∈[-4; -1]=y(-1)=3, min yx∈[-4; -1]=y(-4)=-334.

Ответ: Для отрезка [1;4] – max yx∈[1; 4]=y(2)=3, min yx∈[1; 4]=y(2)=3, для отрезка [-4;-1] – max yx∈[-4; -1]=y(-1)=3, min yx∈[-4; -1]=y(-4)=-334.

См. на рисунке:

Как найти наибольшее и наименьшее значение непрерывной функции на заданном отрезке

Как найти наибольшее и наименьшее значение непрерывной функции на открытом или бесконечном интервале

Перед тем как изучить данный способ, советуем вам повторить, как правильно вычислять односторонний предел и предел на бесконечности, а также узнавать основные методы их нахождения. Чтобы найти наибольшее и/или наименьшее значение функции на открытом или бесконечном интервале, выполняем последовательно следующие действия.

  1. Для начала нужно проверить, будет ли заданный интервал являться подмножеством области определения данной функции.
  2. Определим все точки, которые содержатся в нужном интервале и в которых не существует первой производной. Обычно они бывают у функций, где аргумент заключен в знаке модуля, и у степенных функций с дробно рациональным показателем. Если же эти точки отсутствуют, то можно переходить к следующему шагу.
  3. Теперь определим, какие стационарные точки попадут в заданный промежуток. Сначала приравняем производную к 0, решим уравнение и подберем подходящие корни. Если у нас нет ни одной стационарной точки или они не попадают в заданный интервал, то сразу переходим к дальнейшим действиям.  Их определяет вид интервала.
  • Если интервал имеет вид [a;b), то нам надо вычислить значение функции в точке x=a и односторонний предел limx→b-0f(x).
  • Если интервал имеет вид (a;b], то нам надо вычислить значение функции в точке x=b и односторонний предел limx→a+0f(x).
  • Если интервал имеет вид  (a;b), то нам надо вычислить односторонние пределы limx→b-0f(x),limx→a+0f(x).
  • Если интервал имеет вид [a; +∞), то надо вычислить значение в точке x=a и предел на плюс бесконечности limx→+∞f(x).
  • Если интервал выглядит как (-∞; b], вычисляем значение в точке x=b и предел на минус бесконечности limx→-∞f(x).
  • Если -∞; b, то считаем односторонний предел limx→b-0f(x) и предел на минус бесконечности limx→-∞f(x)
  • Если же -∞; +∞, то считаем пределы на минус и плюс бесконечности limx→+∞f(x),  limx→-∞f(x).
  1. В конце нужно сделать вывод на основе полученных значений функции и пределов. Здесь возможно множество вариантов. Так, если односторонний предел равен минус бесконечности или плюс бесконечности, то сразу понятно, что о наименьшем и наибольшем значении функции сказать ничего нельзя. Ниже мы разберем один типичный пример. Подробные описания помогут вам понять, что к чему. При необходимости можно вернуться к рисункам 4-8 в первой части материала.
Пример 2

Условие: дана функция y=3e1x2+x-6-4. Вычислите ее наибольшее  и наименьшее значение в интервалах  -∞; -4, -∞; -3, (-3;1], (-3;2), [1;2), 2; +∞, [4; +∞).

Решение

Первым делом находим область определения функции. В знаменателе дроби стоит квадратный (квадратичный) трехчлен, который не должен обращаться в 0:

x2+x-6=0D=12-4·1·(-6)=25×1=-1-52=-3×2=-1+52=2⇒D(y): x∈(-∞; -3)∪(-3; 2)∪(2; +∞)

Мы получили область определения функции, к которой принадлежат все указанные в условии интервалы.

Теперь выполним дифференцирование функции и получим:

y’=3e1x2+x-6-4’=3·e1x2+x-6’=3·e1x2+x-6·1×2+x-6’==3·e1x2+x-6·1’·x2+x-6-1·x2+x-6′(x2+x-6)2=-3·(2x+1)·e1x2+x-6×2+x-62

Следовательно, производные функции существуют на всей области ее определения.

Перейдем к нахождению стационарных точек. Производная функции обращается в 0 при x=-12. Это стационарная точка, которая находится в интервалах (-3;1] и (-3;2).

Вычислим значение функции при x=-4 для промежутка (-∞; -4], а также предел на минус бесконечности:

y(-4)=3e1(-4)2+(-4)-6-4=3e16-4≈-0.456limx→-∞3e1x2+x-6=3e0-4=-1

Поскольку 3e16-4>-1, значит, max yx∈(-∞; -4]=y(-4)=3e16-4. Это не дает нам возможности однозначно определяться с наименьшим значением функции. Мы можем только сделать вывод, что внизу есть ограничение -1, поскольку именно к этому значению функция приближается асимптотически на минус бесконечности.

Особенностью второго интервала является то, что в нем нет ни одной стационарной точки и ни одной строгой границы. Следовательно, ни наибольшего, ни наименьшего значения функции мы вычислить не сможем. Определив предел на минус бесконечности и при стремлении аргумента к -3 с левой стороны, мы получим только интервал значений:

limx→-3-03e1x2+x-6-4=limx→-3-03e1(x+3)(x-3)-4=3e1(-3-0+3)(-3-0-2)-4==3e1(+0)-4=3e+∞-4=+∞limx→-∞3e1x2+x-6-4=3e0-4=-1

Значит, значения функции будут расположены в интервале -1; +∞

Чтобы найти наибольшее значение функции в третьем промежутке, определим ее значение в стационарной точке  x=-12, если x=1. Также нам надо будет знать односторонний предел для того случая, когда аргумент стремится к -3 с правой стороны:

y-12=3e1-122+-12-6-4=3e425-4≈-1.444y(1)=3e112+1-6-4≈-1.644limx→-3+03e1x2+x-6-4=limx→-3+03e1(x+3)(x-2)-4=3e1-3+0+3(-3+0-2)-4==3e1(-0)-4=3e-∞-4=3·0-4=-4

У нас получилось, что наибольшее значение функция примет в стационарной точке max yx∈(3; 1]=y-12=3e-425-4. Что касается наименьшего значения, то его мы не можем определить. Все, что нам известно, – это наличие ограничения снизу до -4.

Для интервала (-3;2) возьмем результаты предыдущего вычисления и еще раз подсчитаем, чему равен односторонний предел при стремлении к 2 с левой стороны:

y-12=3e1-122+-12-6-4=3e-425-4≈-1.444limx→-3+03e1x2+x-6-4=-4limx→2-03e1x2+x-6-4=limx→-3+03e1(x+3)(x-2)-4=3e1(2-0+3)(2-0-2)-4==3e1-0-4=3e-∞-4=3·0-4=-4

Значит, max yx∈(-3; 2)=y-12=3e-425-4, а наименьшее значение определить невозможно, и значения функции ограничены снизу числом -4.

Исходя из того, что у нас получилось в двух предыдущих вычислениях, мы можем утверждать, что на интервале [1;2) наибольшее значение функция примет при x=1, а найти наименьшее невозможно.

На промежутке (2; +∞) функция не достигнет ни наибольшего, ни наименьшего значения, т.е. она будет принимать значения из промежутка -1; +∞.

limx→2+03e1x2+x-6-4=limx→-3+03e1(x+3)(x-2)-4=3e1(2+0+3)(2+0-2)-4==3e1(+0)-4=3e+∞-4=+∞limx→+∞3e1x2+x-6-4=3e0-4=-1

Вычислив, чему будет равно значение функции при x=4, выясним, что max yx∈[4; +∞)=y(4)=3e114-4 , и заданная функция на плюс бесконечности будет асимптотически приближаться к прямой y=-1.

Сопоставим то, что у нас получилось в каждом вычислении, с графиком заданной функции. На рисунке асимптоты показаны пунктиром.

Как найти наибольшее и наименьшее значение непрерывной функции на открытом или бесконечном интервале

Это все, что мы  хотели рассказать о нахождении наибольшего и наименьшего значения функции. Те последовательности действий, которые мы привели, помогут сделать необходимые вычисления максимально быстро и просто. Но помните, что зачастую бывает полезно сначала выяснить, на каких промежутках функция будет убывать, а на каких возрастать, после чего можно делать дальнейшие выводы. Так можно более точно определить наибольшее и наименьшее значение функции и обосновать полученные результаты.

Автор статьи

Наталья Игоревна Восковская

Эксперт по предмету «Математика»

Задать вопрос автору статьи

Экстремумы функции

Для того чтобы ввести понятие наибольшего и наименьшего значения функций, вначале познакомимся с таким понятием, как экстремумы функций. Это понятие нам будет необходимо не для самого определения значений таких функций, а для построения схемы нахождения таких промежутков для конкретно заданных функций.

Определение 1

Точка $x’$ входящая в область определения функции называется точкой экстремума, если она либо будет точкой максимума, либо будет точкой минимума для функции $f(x)$.

Определение 2

Точка $x’$ будет называться точкой максимума для введенной функции $f(x)$, если у она имеет такую окрестность, что для всех точек $x$, которые входят в эту окрестность, будет верно $f(x)le f(x'{rm })$.

Логотип baranka

Сдай на права пока
учишься в ВУЗе

Вся теория в удобном приложении. Выбери инструктора и начни заниматься!

Получить скидку 3 000 ₽

Определение 3

Точка $x_0$ будет называться точкой минимума для введенной функции $f(x)$, если она имеет такую окрестность, что для всех точек $x$, которые входят в эту окрестность, будет верно $f(x)ge f(x'{rm })$.

Чтобы полностью разобраться в данном понятии, далее введем понятие критической точки функции.

Определение 4

Точка $x’$ будет называться критической точкой для данной функции $f(x)$, если выполняются два следующих условия:

  1. Точка $x’$ является внутренней точкой для области определения данной функции;
  2. $f’left(x'{rm }right)=0$ или не существует.

Сформулируем без доказательства теоремы о необходимом (теорема 1) и достаточном (теорема 2) условии для существования точки экстремума.

Если $y=f(x)$ имеет экстремум в точке $x_0$, то либо её производная в ней равняется нулю, либо производная в ней не существует.

«Точки экстремума, наибольшее и наименьшее значение на промежутке» 👇

Теорема 2

Пусть точка $x’$ будет критической для $y=f(x)$ и принадлежит интервалу $(a,b)$, причем на каждом интервале $left(a,x'{rm }right) и (x'{rm },b)$ производная $f'(x)$ существует и сохраняет один и тот же знак. В этом случае:

  1. Если в $(a,x'{rm })$ $f’left(xright) >0$, а в $(x'{rm },b)$ $f’left(xright)
  2. Если в $(a,x'{rm })$ $f’left(xright)0$, то $x’$ –будет точкой минимума для этой функции.
  3. Если и в $(a,x'{rm })$, и в $(x'{rm },b)$ производная $имеет один и тот же постоянный знак$, то $x’$ не будет точкой экстремума для этой функции.

На рисунке 1 мы можем наглядно увидеть смысл теоремы 2.

Рисунок 1.

Примеры точек экстремумов вы можете видеть на рисунке 2.

Рисунок 2.

Правило исследования на экстремум

  1. Найти $D(f)$;
  2. Найти $f'(x)$;
  3. Найти точки, где $f’left(xright)=0$;
  4. Найти точки, где $f'(x)$ не будет существовать;
  5. Отметить на координатной прямой $D(f)$ и все найденные в 3 и 4 пункте точки;
  6. Определить знак $f'(x)$ на полученных промежутках;
  7. Используя теорему 2, сделать заключение по поводу всех найденных точек.

Понятие наибольшего и наименьшего значений

Определение 5

Функция $y=f(x)$, которая имеет областью определения множество $X$, имеет наибольшее значение в точке $x’in X$, если выполняется

[fleft(xright)le f(x’)]

Определение 6

Функция $y=f(x)$, которая имеет областью определения множество $X$, имеет наименьшее значение в точке $x’in X$, если выполняется

[fleft(xright)ge f(x’)]

Чтобы найти наименьшее и наибольшее значение заданной функции на каком либо отрезке необходимо произвести следующие действия:

  1. Найти $f'(x)$;
  2. Найти точки, в которых $f’left(xright)=0$;
  3. Найти точки, в которых $f'(x)$ не будет существовать;
  4. Выкинуть из точек, найденных в пунктах 2 и 3 те, которые не лежат в отрезке $[a,b]$;
  5. Вычислить значения в оставшихся точках и на концах $[a,b]$;
  6. Выбрать из этих значений наибольшее и наименьшее.

Примеры задач

Пример 1

Найти наибольшее и наименьшее значения на [0,6]: $fleft(xright)=x^3-3x^2-45x+225$

Решение.

  1. $f’left(xright)=3x^2-6x-45$;
  2. $f’left(xright)=0$;
  3. [3x^2-6x-45=0]
  4. [x^2-2x-15=0]
  5. [x=5, x=-3]
  6. $f'(x)$ существует на всей $D(f)$;
  7. $5in left[0,6right]$;
  8. Значения:

    [fleft(0right)=225] [fleft(5right)=50] [fleft(6right)=63]

  9. Наибольшее значение равняется $225$, наименьшее равняется $50.$

Ответ: $max=225, min=50$.

Пример 2

Найти наибольшее и наименьшее значения на [-1,1]:$fleft(xright)=frac{x^2-4x+4}{x-2}$

Решение.

[fleft(xright)=frac{x^2-4x+4}{x-2}=frac{{(x-2)}^2}{x-2}=x-2, xne 2]

  1. $f’left(xright)=(x-2)’=1$;

    Точек экстремума нет.

  2. Значения:

    [fleft(-1right)=-3] [fleft(1right)=-1]

Ответ: $max=-1, min=-3$.

Находи статьи и создавай свой список литературы по ГОСТу

Поиск по теме

Задание 11 первой части Профильного ЕГЭ по математике — это нахождение точек максимума и минимума функции, а также наибольших и наименьших значений функции с помощью производной.

Вот какие типы задач могут встретиться в этом задании:

Нахождение точек максимума и минимума функций

Исследование сложных функций

Нахождение наибольших и наименьших значений функций на отрезке

Нахождение точек максимума и минимума функций

1. Найдите точку максимума функции displaystyle y=-{{x^2+289}over{x}}.

Найдем производную функции.

Приравняем производную к нулю. Получим:

x^2=289Leftrightarrow left[ begin{array}{c}  x=17, hfill \ x=-17. end{array} right.

Исследуем знаки производной.

В точке x = 17 производная y меняет знак с «плюса» на «минус». Значит, x= 17 — точка максимума функции y(x).

Ответ: 17.

2. Найдите точку минимума функции y=2x^2-5x+lnx-3.

Найдем производную функции.

y{

Приравняем производную к нулю.

4x-5+{{1}over{x}}=0Leftrightarrow 4x^2-5x+1=0Leftrightarrow left[ begin{array}{c}  x=1, \ x={{1}over{4}}. end{array} right.

Определим знаки производной.

В точке x = 1 производная y меняет знак с «минуса» на «плюс». Значит, x= 1 — точка минимума функции y(x).

Ответ: 1.

Исследование сложных функций

3. Найдите точку максимума функции y=2^{5-8x-x^2}.

Перед нами сложная функция y=2^{5-8x-x^2}. Возможно, вы знаете формулы производной сложной функции. Но вообще-то их изучают на первом курсе вуза, поэтому мы решим задачу более простым способом.

Так как функция y=2^t монотонно возрастает, точка максимума функции y=2^{5-8x-x^2} будет при том же x_0, что и точка максимума функции tleft(xright)=5-8x-x^2. А ее найти легко.

t^{

t^{ при x=-4. В точке x = -4 производная {{ t}}^{{ меняет знак с «плюса» на «минус». Значит, x= - 4 — точка максимума функции { t}left({ x}right).

Заметим, что точку максимума функции tleft(xright)=5-8x-x^2 можно найти и без производной.

Графиком функции tleft(xright) является парабола ветвями вниз, и наибольшее значение tleft(xright) достигается в вершине параболы, то есть при x=-frac{8}{2}=-4.

Ответ: – 4.

4. Найдите абсциссу точки максимума функции y=sqrt{4-4x-x^2}.

Напомним, что абсцисса — это координата по X.

Снова сложная функция. Применяем тот же прием, что и в предыдущей задаче.

Так как функция y=sqrt{z} монотонно возрастает, точка максимума функции y=sqrt{4-4x-x^2} является и точкой максимума функции tleft(xright)=4-4x-x^2.

Это вершина квадратичной параболы tleft(xright)=4-4x-x^2;x_0=frac{-4}{2}=-2.

Нахождение наибольших и наименьших значений функций на отрезке

5. Найдите наибольшее значение функции y=x^3+2x^2-4x+4 на отрезке [-2;0].

Мы помним, что наибольшее значение функции на отрезке может достигаться либо в точке максимума, либо на конце отрезка. Эти случаи показаны на рисунке.

Будем искать точку максимума функции y=x^3+2x^2-4x+4 с помощью производной. Найдем производную и приравняем ее к нулю.

y

y

{3x}^2+4x-4=0;

D=64;x=frac{-4pm 8}{6};x_1=frac{2}{3},x_2=-2.

Найдем знаки производной.

В точке x = - 2 производная равна нулю и меняет знак с “+” на “-“. Значит, x = – 2 — точка максимума функции y(x). Поскольку при xin [-2;0] функция y(x) убывает, y_{max}left(xright)=yleft(-2right)=12. В этой задаче значение функции на концах отрезка искать не нужно.

Ответ: 12.

6. Найдите наименьшее значение функции y={4x}^2-10x+2lnx-5 на отрезке [0,3;3].

Найдем производную функции y={4x}^2-10x+2lnx-5 и приравняем ее к нулю.

y при x_1=1,x_2=frac{1}{4}.

Найдем знаки производной.

Точка x_1=1 — точка минимума функции yleft(xright). Точка x_2=frac{1}{4} не лежит на отрезке [0,3;1]. Поэтому

 и  Значит, наименьшее значение функции на отрезке left[0,3;1right] достигается при x=1. Найдем это значение.

y_{min}left(xright)=yleft(1right)=4-10-5=-11.

Ответ: -11.

7. Найдите наименьшее значение функции y=9x-{ln left(9xright)}+3 на отрезке left[frac{1}{18};frac{5}{18}right].

Иногда перед тем, как взять производную, формулу функции полезно упростить.

y=9x-{ln left(9xright)}+3=9x-{ln 9-{ln x}}+3.

Мы применили формулу для логарифма произведения. y при x=frac{1}{9}.

Если  то  Если , то 

Значит, x=frac{1}{9} — точка минимума функции y(x). В этой точке и достигается наименьшее значение функции на отрезке left[frac{1}{18};frac{5}{18}right].

y_{min}left(xright)=yleft(frac{1}{2}right)=1+3=4.

Ответ: 4.

8. Найдите наибольшее значение функции y(x)=14x-7tgx-3,5pi +11 на отрезке left[-frac{pi }{3};frac{pi }{3}right].

Найдем производную функции y(x)=14x-7tgx-3,5pi +11. y

Приравняем производную к нулю: 14-frac{7}{{cos}^2x}=0.

{cos}^2x=frac{1}{2}.

{cos}^2x=pm frac{1}{sqrt{2}}=pm frac{sqrt{2}}{2}. Поскольку xin left[-frac{pi }{3};frac{pi }{3}right], y если x=pm frac{pi }{4}.

Найдем знаки производной на отрезке left[-frac{pi }{3};frac{pi }{3}right].

При x=frac{pi }{4} знак производной меняется с «плюса» на «минус». Значит, x=frac{pi }{4} — точка максимума функции y(x).

Мы нашли точку максимума, но это еще не все. Сравним значения функции в точке максимума и на конце отрезка, то есть при x=-frac{pi }{3} и x =frac{pi }{4}.

yleft(frac{pi }{4}right)=-7+11=4;

Мы нашли, что y_{max}left(xright)=yleft(frac{pi }{4}right)=-7+11=4.

Заметим, что если вам попадется такая задача в первой части ЕГЭ по математике, то находить значение функции при -frac{pi }{3} не обязательно. Как мы видим, это значение — число иррациональное. А в первой части ЕГЭ по математике ответом может быть только целое число или конечная десятичная дробь.

Ответ: 4.

9. Найдите наименьшее значение функции y=e^{2x}-{8e}^x+9 на отрезке [0;2].

Снова сложная функция. Запишем полезные формулы:

{{(e}^{-x})}^{

{left(e^{cx}right)}^{

{(e}^{x+a})

Найдем производную функции y=e^{2x}-{8e}^x+9.

y

y если e^x=4. Тогда x=ln4.

 При x=ln4 знак производной меняется с «минуса» на «плюс». Значит, x=ln4 — точка минимума функции y(x). yleft(ln4right)=4^2-8cdot 4+9=16-32+9=-7.

Ответ: -7.

10. Найдите наибольшее значение функции y=12cosx+6sqrt{3}x-2sqrt{3}pi +6 на отрезке left[0;frac{pi }{2}.right]

Как всегда, возьмем производную функции и приравняем ее к нулю.

y

y 12sinx=6sqrt{3};

sinx=frac{sqrt{3}}{2}.

По условию, xin left[0;frac{pi }{2}right]. На этом отрезке условие sinx=frac{sqrt{3}}{2} выполняется только для x=frac{pi }{3}. Найдем знаки производной слева и справа от точки x=frac{pi }{3}.

В точке x_0=frac{pi }{3} производная функции меняет знак с «плюса» на «минус». Значит, точка x_0=frac{pi }{3} — точка максимума функции y(x). Других точек экстремума на отрезке left[0;frac{pi }{2}right] функция не имеет, и наибольшее значение функции { y=12cosx+6}sqrt{{ 3}}{ }{ x}{ -}{ 2}sqrt{{ 3}}{ }pi { +6} на отрезке left[{ 0};frac{pi }{{ 2}}right] достигается при { x=}frac{pi }{{ 3}}.

y_{max}left(xright)=yleft(frac{pi }{3}right)=12.

Ответ: 12.

11.Найдите наименьшее значение функции y=16x-6sinx+6 на отрезке left[0;frac{pi }{2}right].

Найдем производную функции и приравняем ее к нулю.  — нет решений.

Что это значит? Производная функции y=16x-6sinx+6 не равна нулю ни в какой точке. Это значит, что знак производной в любой точке одинаков, а функция не имеет экстремумов и является монотонной.

Поскольку cosxle 1, получим, что  для всех x, и функция yleft(xright)=16x-6sinx+6 монотонно возрастает при xin left[0;frac{pi }{2}right].

Значит, наименьшее свое значение функция принимает в левом конце отрезка left[{ 0};frac{pi }{{ 2}}right], то есть при x=0.

y_{min}left(xright)=yleft(0right)=6.

Ответ: 6

Благодарим за то, что пользуйтесь нашими статьями.
Информация на странице «Задание 11 Профильного ЕГЭ по математике» подготовлена нашими авторами специально, чтобы помочь вам в освоении предмета и подготовке к ЕГЭ и ОГЭ.
Чтобы успешно сдать необходимые и поступить в ВУЗ или техникум нужно использовать все инструменты: учеба, контрольные, олимпиады, онлайн-лекции, видеоуроки, сборники заданий.
Также вы можете воспользоваться другими материалами из данного раздела.

Публикация обновлена:
08.05.2023

На этой странице вы узнаете

  • Кто всегда протянет руку помощи в определении производной?
  • Что такое сложная функция и зачем тут матрешка?
  • Как никогда не ошибаться при решении задач с производными?

Теория теорией, а дифференцировать хочется всегда. Эта статья посвящена практике нахождения производных.

Производные основных функций

Должно быть, вы уже слышали о производной и даже пробовали взять её мозговым штурмом. При отрицательном ответе вам обязательно нужно прокатиться на американских горках в нашей статье «Производная». В ней рассмотрели основные понятия производной.

Главный вопрос этой статьи: как ее находить? Для этого существуют свои формулы и правила, которых необходимо придерживаться для правильного решения заданий. 

Ниже приведена таблица с формулами для нахождения производных основных функций. Применяя эти формулы, можно найти производную почти любой функции. 

Не пугайтесь, если вам покажется, что их много: это основные формулы, с помощью которых можно решить большинство задач.

1 C’ = 0, C = const
2 ((x^n)’ = n * x^{n — 1}, x > 0)
3 ((a^x)’ = a^x * ln(a), a > 0, a neq 1)
4 ((e^x)’ = e^x)
5 ((log_{a}x)’ = frac{1}{x * ln(a)}, x > 0, a > 0, a neq 1)
6 ((ln(x))’ = frac{1}{x}, x > 0)
7 ((sqrt{x})’ = frac{1}{2sqrt{x}}, x > 0)
8 (sin(x))’ = cos(x)
9 (cos(x))’ = -sin(x)
10 ((tg(x))’ = frac{1}{cos^{2}x}, x neq frac{pi}{2} + pi n, n in Z)
11 ((ctg(x))’ = -frac{1}{sin^{2}x}, x neq pi n, n in Z)

Смотреть на формулы и учить их — это круто, прямо ощущаем себя великими учеными. Что может быть круче этого? Только применять их на практике. Рассмотрим несколько примеров нахождения производной. 

Пример 1. Найдите производную функции f(x) = 5. 

Решение: 5 — это число, то есть константа. Тогда, пользуясь первой формулой в таблице, получаем:

f'(x) = 5′ = 0. 

Ответ: 0

Пример 2. Найдите производную функции (f(x) = x^4)

Решение: В этом случае необходимо воспользоваться второй формулой из таблицы. 

(f'(x) = (x^4)’ = 4 * x^{4-1} = 4 * x^3)

Ответ: (4x^3)

Пример 3. Найдите производную функции (f(x) = e^x)

Решение: В этом случае необходимо воспользоваться четвертой формулой из таблицы. 

(f'(x) = (e^x)’ = e^x)

Ответ: (e^x)

Правила дифференцирования

С полной уверенностью можем сказать, что вам  встречались сложные функции. Даже намного сложнее, чем те, которые приведены в таблицах. Там и сумма, и произведение, и формула в формуле. Одним словом: ужас! Как брать производную, если перед функцией стоит коэффициент, или в функцию включено несколько разных выражений? На этот случай существуют правила дифференцирования. 

Кто всегда протянет руку помощи в определении производной?

В сложных функциях невозможно пользоваться только формулами для нахождения производной.

Если функция
— усложнена коэффициентом, 
— представлена в виде суммы, произведения или частного 
— или является сложной функцией, 
то для выбора правильной производной необходимо воспользоваться правилами дифференцирования. Они играют роль супергероев от мира производных. Рассмотрим их внимательнее.

1. Коэффициент можно вынести за знак производной. 

(k * f(x))’ = k * (f(x))’

Например, необходимо взять производную у функции f(x) = 6sin(x). Тогда, пользуясь правилом дифференцирования и таблицей, получаем ответ 6cos(x). 

2. Производная суммы (разности) равняется сумме (разности) производных. 

((f(x) pm g(x))’ = f'(x) pm g'(x))

Найдем производную (f(x) = 4x^5 — sqrt{x} + cos(x)).

(f'(x) = (4x^5 — sqrt{x} + cos(x))’ = (4x^5)’ — (sqrt{x})’ + (cos(x))’ = 4 * 5 * x^{5 — 1} — frac{1}{2sqrt{x}} — sin(x))
(f'(x) = 20x^4 — frac{1}{2sqrt{x}} — sin(x). )

3. Производная произведения. 

(f(x) * g(x))’ = f'(x) * g(x) + f(x) * g'(x)

Для примера возьмем производную функции f(x) = x2 * ln(x)

f'(x) = (x2 * ln(x))’ = (x2)’ * ln(x) + x2 * (ln(x))’
(f'(x) = 2x * ln(x) + x^2 * frac{1}{x} = 2x * ln(x) + x)

4. Производная частного. 

((frac{f(x)}{g(x)})’ = frac{f'(x) * g(x) — f(x) * g'(x)}{g^{2}(x)})

Возьмем производную функции (f(x) = frac{e^x}{3x})

(f'(x) = frac{(e^x)’ * 3x — ex * (3x)’}{(3x)^2} = frac{e^x * 3x — e^x * 3}{9x^2} = frac{3e^x * (x-1)}{9x^2} = frac{e^x * (x-1)}{3x^2})

5. Производная сложной функции. 

Сложная функция — это функция, внутри которой есть другая функция. 

Что такое сложная функция и зачем тут матрешка?

Давайте представим матрешку: в одну большую куклу складывается куколка поменьше, а в нее еще меньше и так далее. Точно так же и с функцией: “внутри” одной функции может лежать другая функция. 

Например, у нас есть две функции: (sqrt{x}) и cos(x). А теперь попробуем поместить корень в функцию с косинусом, и получим (cos(sqrt{x})). Это и будет сложная функция.

Чтобы найти производную сложной функции, необходимо найти производную “внутренней” функции и умножить ее на производную “внешней” функции. 

(f(g(x))’ = g'(x) * f'(g(x))

Найдем производную уже рассмотренной функции (f(x) = cos(sqrt{x})). 

(f'(x) = (cos(sqrt{x}))’ = (sqrt{x})’ * (cos(sqrt{x}))’ = frac{1}{2sqrt{x}} * (-sin(sqrt{x})) = -frac{sin(sqrt{x})}{2sqrt{x}})

Исследование функции с помощью производной 

В задании нам может быть дана только функция без ее графика. Что делать в таком случае, если нам нужно найти, например, отрезки возрастания, точки экстремума, наибольшее или наименьшее значение функции? Не во всех случаях получится построить график, да и это займет достаточно большое количество времени, которое и без того ограничено на экзамене. 

В этом случае мы можем проанализировать поведение функции с помощью производной. 

Исследуем функцию f(x) = (x — 4)2(x + 11) + 4. 

Cначала возьмем производную от этой функции: 

f'(x) = ((x — 4)2(x + 11))+ 4′ = ((x — 4)2(x + 11))’ = ((x — 4)2)'(x + 11) + (x — 4)2(x + 11)’
f'(x) = 2(x — 4)(x + 11) + (x — 4)2 * 1 = (x — 4)(2(x + 11) + (x — 4)) = (x — 4)(3x + 18)

Любое исследование функции с помощью производной начинается именно с дифференцирования функции. 

Теперь рассмотрим алгоритм нахождения точек минимума и максимума:

1 шаг. Нужно найти производную функции.

2 шаг. Найденную производную необходимо приравнять к 0 и решить полученное уравнение. 

3 шаг. Расставить корни полученного уравнения на числовой прямой. 

4 шаг. Определяем знаки производной на промежутках. Для этого необходимо подставить любое значение с выбранного промежутка в производную функции. 

5 шаг. Определить, какие точки будут точками минимума (в них знак меняется с минуса на плюс), а какие — точками максимума (знак меняется с плюса на минус). 

Найдем точки минимума и максимума в нашей функции. Поскольку производную мы уже взяли, можно сразу перейти ко второму шагу: 

(x — 4)(3x + 18) = 0
x = 4, x = -6.

Полученные значения х расставляем на числовой прямой: 

Теперь определим знаки на промежутках слева направо.

1. Возьмем точку -10 и подставим ее в производную функции: 
(-10 — 4)(3 * (-10) + 18) = (-14) * (-12) = 168. Производная на этом промежутке будет положительной. 

2. Возьмем точку 0 и подставим ее в производную функции: 
(0 — 4)(3 * 0 + 18) = (-4) * 18 = -72. Производная на этом промежутке будет отрицательной. 

3. Возьмем точку 5 и подставим ее в производную функции: 
(5 — 4)(3 * 5 + 18) = 33. Производная на этом промежутке будет положительной. 

Расставим полученные знаки на прямой: 

Остался последний пятый шаг. В точке -6 производная меняет знак с плюса на минус, значит, это точка максимума. В точке 4 производная меняет знак с минуса на плюс, значит, это точка минимума. 

Важно!
Если в задании встречается формулировка “Найдите точку минимума (максимума) функции”, то необходимо пользоваться именно этим алгоритмом. 

Но это не все выводы, которые уже можно сделать о функции. Вспомним, что функция возрастает, когда производная положительна, а убывает, когда производная отрицательна. Поскольку мы уже определили знаки производной, то смело можем сделать вывод, что на промежутках до -6 и после 4 функция будет возрастать, а на промежутке от -6 до 4 — убывать. 

Однако могут встретиться задания, в которых необходимо найти наибольшее или наименьшее значение функции на определенном интервале. 

Для выполнения таких заданий существует следующий алгоритм нахождения наибольшего и наименьшего значения функции.

Шаг 1. Найти производную функции. 

Шаг 2. Найти точки минимума и максимума функции.

Шаг 3. Определить, какие из точек минимума и максимума принадлежат заданному интервалу. 

Шаг 4. Найти значение функции в отобранных в предыдущем шаге точках, а также в точках, которые являются границами заданного интервала. Для этого необходимо подставить точки в функцию (не в производную от функции).

Для примера найдем наибольшее значение функции f(x) = (x — 4)2(x + 11) + 4 на отрезке [-10; 0]. 

Первые два шага мы уже выполнили, когда рассматривали алгоритм нахождения точек минимума и максимума. Из них отрезку [-10; 0] принадлежит х = -6 — точка максимума. 

Теперь определим значение функции в трех точках: 

f(-10) = (-10 — 4)2(-10 + 11) + 4 = 196 + 4 = 200
f(-6) = (-6 — 4)2(-6 + 11) + 4 = 500 + 4 = 504
f(0) = (0 — 4)2(0 + 11) + 4 = 176 + 4 = 180

Наибольшее из полученных значений — это 504. Это и будет ответ. 

Как никогда не ошибаться при решении задач с производными?

Может возникнуть вопрос, почему важно проверять значение функции и на границах отрезка? В заданиях ЕГЭ очень часто встречаются случаи, когда нужно найти наибольшее значение, и в интервале лежит точка максимума, или когда нужно найти наименьшее значение функции и в интервале лежит точка минимума. Логично будет проверить только экстремумы, поскольку в них, скорее всего, достигается наибольшее или наименьшее значение. 

Однако стоит вспомнить, что мы не видим график функции и не можем с точностью определить, что в экстремуме достигается нужное нам значение. С помощью экстремумов мы можем описать поведение функции: где она возрастает, а где убывает. Но можно столкнуться с графиком, на котором граничная точка будет лежать выше или ниже точки экстремума. Тогда наибольшее или наименьшее значение будет достигаться именно в ней. Пример на картинке (красными линиями обозначены границы отрезка). 

Подведем итог.
Как можно исследовать функцию с помощью производной?
С помощью производной можно с точностью сказать, на каких участках функция будет возрастать и убывать, сколько точек максимума и минимума у нее есть, какое наибольшее или наименьшее значение принимает функция на заданном участке. 

Фактчек

  • Для нахождения производной необходимо пользоваться специальными формулами для производной. С их помощью можно найти производную любой из основных функций.
  • Если функция усложнена коэффициентом, является сложной или представлена в виде суммы, произведения или частного, то необходимо пользоваться правилами дифференцирования. Они помогут правильно найти производную.
  • Сложная функция — это функция, внутри которой есть другая функция. 
  • С помощью производной можно исследовать функцию, а именно найти точки минимума и максимума, определить, на каких участках функция возрастает и убывает, найти наибольшее и наименьшее значение функции на заданном отрезке. 

Проверь себя

Задание 1.
Чему будет равна производная f(x) = 3?

  1. 3;
  2. 1;
  3. 0;
  4. Производную этой функции невозможно найти.

Задание 2. 
Чему будет равна производная f(x) = 5x2?

  1. 10x;
  2. 10x2;
  3. 5x2;
  4. 2x.

Задание 3.
Чему будет равна производная f(x) = 13x + 5 + x3?

  1. 18 + 3x2;
  2. 13 + 3x2;
  3. 18;
  4. 3x2.

Задание 4.
Чему будет равна производная f(x) = ln(x)?

  1. x
  2. (frac{1}{x})
  3. (frac{1}{2sqrt{x}})
  4. ex

Задание 5.
Чему будет равна производная f(x) = tg(x)?

  1. (frac{1}{cos^{2}(x)})
  2. (-frac{1}{sin^{2}(x)})
  3. (-frac{1}{cos^{2}(x)})
  4. (frac{1}{sin^{2}(x)})

Ответы: 1. — 3 2. — 1 3. — 2 4. — 2 5. — 1

государственное
образовательное учреждение дополнительного профессионального образования
(повышения квалификации) специалистов «Кузбасский региональный институт
повышения квалификации и переподготовки работников образования»

Факультет повышения квалификации

Кафедра естественнонаучных и математических дисциплин

Решение заданий на нахождение
наибольших и наименьших значений и точек экстремума функции

(методические рекомендации).

Итоговая работа

Срок обучения с 01. 03 2016
г. по 18.03. 2016 г.

Исполнитель:

Сипкова Елена Васильевна,

 учитель математики

МОУ Белогорская средняя общеобразовательная школа

(lenasipkova@mail.ru)

 

Кемерово 2016

Содержание

Введение ……………………………………………………………………………………………..3

1.     Методические рекомендации
обучения учащихся решению заданий 

по   нахождению
наибольшего и наименьшего значения функции с

помощью
производной……………
…………………………………………5

2.     Методические рекомендации
обучения учащихся решению заданий вычислению точек экстремума……………..……………………………..24

Заключение………………………………………………………………………41

Литература……………………………………………………………………42

Приложения………………………………………………….. …………………………..43

 

Введение

Содержание
разделов курса, составляющих начала математического анализа вызывает
затруднение у многих обучающихся

Однако, все
обучающиеся должны знать определение производной, основные правила
дифференцирования и формулы производных элементарных функций, понимать
геометрический смысл производной, знать уравнение касательной.

Первоначально
происходит формирование начальных умений находить производные элементарных
функций на основе определения производной.

В дальнейшем все
обучающиеся должны овладеть правилами дифференцирования суммы, произведения и
частного двух функций, вынесения постоянного множителя за знак производной.

В результате
изучения темы у обучающихся должны быть сформированы умения находить
производные элементарных функций.

После усвоения
обучающимися таблиц производных элементарных функций и правил дифференцирования
происходит знакомство с геометрическим смыслом производной, бучение составлению
уравнений касательной к графику функции в заданной точке.

Очень важно знать
всем обучающимся, что с помощью производной можно аналитически установить много
важных свойств функции. Все обучающиеся должны по графику функции выявлять ее
промежутки возрастания и убывания. Находить интервалы монотонности функции,
заданной аналитически.

В ходе изучения
темы необходимо все знать определения точек максимума и минимума, стационарных
и критических точек; уметь применять необходимые и достаточные условия
экстремума для нахождения точек экстремума функции.

Следующей целью
ставиться научиться строить график функции. С помощью производной, применять
производную к нахождению наибольшего и наименьшего значения функций, решать
прикладные задачи на «Экстремум»

Задания на
выполнение производной стали традиционными на ЕГЭ по математике. Они
предназначены для формирования устойчивых навыков решения задач на исследование
функций с применением производной.

Эти задания, по
статистике ЕГЭ прошлых лет, относятся к наиболее проблемным для обучающихся,
поэтому целью своей работы я взяла «алгоритмизацию» процессов обучения по
данной теме.

Воспользовавшись
методическими указаниями обучающиеся сэкономят время для решения задач.

Основная цель моей
работы – повысить уровень математического образования выпускников, что дает им
возможность остановиться на решении не только доступных задач, но и интересных
новых заданий.

В  работе приведены
задачи с готовым решением, однако обучающиеся рекомендуют решать задачи
самостоятельно и сравнивать собственное решение с приведенным в работе.

1.        
Методические
рекомендации обучения учащихся решению заданий по   нахождению наибольшего и
наименьшего значения функции с помощью производной

Формирование
умения решать задачи на нахождение наибольших и наименьших значений – одна из
самых важных целей изучения математического анализа в школе. Решение задач
этого типа, основанное на применении производной, имеет большую прикладную
направленность.

Пусть функция = f (x) непрерывна на отрезке [a; b]. В этом случае, как известно, она
принимает как наибольшее, так и наименьшее значения на этом отрезке. Во многих
прикладных вопросах важно найти те точки отрезка [
ab], которым отвечают наибольшее и
наименьшее значения функции.

При решении этой
задачи возможны два случая:

1)  либо наибольшее (наименьшее) значение
функции достигается внутри отрезка, и тогда эти значения окажутся в числе
экстремумов функции;

2)  либо наибольшее (наименьшее) значение
достигается на концах отрезка [
ab].

Итак, чтобы найти
наибольшее и наименьшее значения непрерывной на отрезке функции
y = f (x), достаточно:

1. Найти все критические точки,
принадлежащие [
ab], и вычислить значения функции в
этих точках.

2. Вычислить значения функции на концах
отрезка [
ab], то есть найти f (a) и f (b).

3. Сравнить полученные результаты:
наибольшее из найденных значений является наибольшим значением функции на
отрезке [
ab]; аналогично, наименьшее из
найденных значений является наименьшим значение функции на этом отрезке.

Замечание 1. При нахождении критических
точек можно использовать соображения геометрического характера, изобразив
схематически график функции.

Замечание 2. Отыскание наибольшего и
наименьшего значений функции можно упростить, если воспользоваться следующими
свойствами непрерывных функций:

1)                      
если
функция
y = f (x) на
отрезке [
a; b] непрерывна и возрастает, то m = f (a) и M = f (b);

2)                      
если
функция
y = f (x) на
отрезке [
a; b] непрерывна и убывает, то m = f (b) и M = f (a);

3)                      
если
функция
y = f (x),
непрерывная на отрезке [
a; b], имеет на этом отрезке только одну
точку максимума
x0 (и ни одной точки минимума), то
наибольшее значение на данном отрезке есть
M = f (x0);

4)                      
если
функция
y = f (x),
непрерывная на отрезке [
a; b], имеет на этом отрезке только одну
точку минимума
x0 (и ни одной точки максимума), то
наименьшее значение на данном отрезке есть
m = f (x0).

Алгоритм решения задач на нахождение наибольшего или
наименьшего значения функции:

Если
в задаче требуется найти максимальное или минимальное значение
функции f(x) на отрезке [a; b], выполняем следующие
действия:

1.    
Найти производную функции: f ‘(x).

2.    
Решить уравнение f”(x) = 0. Если корней нет,
пропускаем третий шаг и переходим сразу к четвертому.

3.    
Из полученного набора корней вычеркнуть все, что лежит
за пределами отрезка [a; b]. Оставшиеся числа обозначим x1, x2, …, xn —
их, как правило, будет немного.

4.    
Подставим концы отрезка [a; b] и точки x1, x2, …, xn
в исходную функцию. Получим набор чисел f(a), f(b), f(x1), f(x2), …, f(xn),
из которого выбираем наибольше или наименьшее значение —
это и будет ответ.

Небольшое
пояснение по поводу вычеркивания корней, когда они совпадают
с концами отрезка. Их тоже можно вычеркнуть, поскольку
на четвертом шаге концы отрезка все равно подставляются
в функцию — даже если уравнение

f
‘(x) = 0 не имело решений.

Также следует внимательно
читать условие задачи. Когда требуется найти значение функции (максимальное
или минимальное), концы отрезка и точки x1, x2, …, xn
подставляются именно в функцию, а не в ее производную.

Задача 1. Найти наибольшее значение функции
y = x3 + 3x2 − 9x – 7
на отрезке [−5; 0].

Решение. Для начала найдем производную: y’ = (x3 + 3x2 − 9x − 7)’ =
3x2 + 6x − 9.

Затем решаем уравнение: y’ = 0  3x2 + 6x − 9 = 0  … 
x = −3; x = 1.

Вычеркиваем корень x = 1, потому что
он не принадлежит отрезку [−5; 0].

Осталось вычислить значение функции на концах отрезка
и в точке x = −3:
y(−5) = (−5)3 + 4·(−5)2 − 9·(−5) − 7 = −12;
y(−3) = (−3)3 + 4·(−3)2 − 9·(−3) − 7 = 20;
y(0) = 03 + 4·02 − 9·0 − 7 = −7.

Очевидно, наибольшее значение равно 20 — оно достигается
в точке x = −3.

Ответ: 20

Задача 2. Найти наибольшее и
наименьшее значения функции

f (x) = 2x3 – 6x + 5 на отрезке .

Решение. 1. Находим критические
точки, принадлежащие :

f¢ (x) = 6x2 – 6 = 6(x2 – 1), 6(x2 – 1) = 0, x1 = –1, x2 = 1.

Вычислим значения
функции в этих точках:

f (–1) = 2 × (–1)3 – 6 × (–1) + 5 = 9;     f (1) = 2 × 13 – 6 × 1 + 5 = 1.

2. Вычислим
значения функции на концах отрезка:

3. Таким образом, наибольшее значение
данной функции на рассматриваемом отрезке есть
f (–1) = 9, а наименьшее

Ответ: f (–1) = 9,

Дробно –
рациональные функции.

Задача 3.  Найдите
наименьшее значение функции
  на отрезке [1; 9].

Решение: Найдём производную данной
функции: . Приведём полученное выражение к общему
знаменателю и разложим числитель на множители: .

Отрезку [1; 9]
принадлежит точка х=6, в которой производная меняет знак с минуса на
плюс. Таким образом , точка х=6 является точкой минимума и единственной
точкой экстремума на данном отрезке. Значит, своего наименьшего значения на
данном отрезке функция достигает именно в этой точке. Найдём наименьшее
значение:

Ответ: 12.

Задача 4. Найдите
наименьшее значение функции на отрезке:  у =  
  на  отрезке
[1;14].

Решение. Заметим, что функция не определена в точке 0.

Берем производную дроби:

B15_22

Приравниваем
производную к нулю и отыскиваем корни:

B15_23

Один из корней нас не интересует, так как промежутку не
принадлежит, а во второй точке производная меняет знак с отрицательного на
положительный. То есть функция имеет минимум в данной точке. Определим ее
минимальное значение:

B15_24    Ответ: 4.

Целые рациональные функции

Задача 5. Найдите наибольшее значение
функции y=x^3-3x+4 на
отрезке [-2;0].

Решение. Найдём производную заданной функции:

Найти наибольшее значение функции у=х^3-3^x+4

Найдем нули
производной:

http://matematikalegko.ru/wp-content/uploads/2014/06/21.gif

Указанному в
условии интервалу принадлежит точка х = –1.

Вычисляем значения
функции в точках   –2, –1 и 0:

http://matematikalegko.ru/wp-content/uploads/2014/06/31.gif

Наибольшее
значение функции равно 6.

Ответ: 6

Задача
6
. Найдите наибольшее значение функции у = х3 – 6х2   на отрезке

[–3;3].  

Решение. Найдём производную заданной функции:

http://matematikalegko.ru/wp-content/uploads/2014/06/71.gif

Найдем нули
производной:

http://matematikalegko.ru/wp-content/uploads/2014/06/81.gif

Указанному в
условии интервалу принадлежит точка х = 0.

Вычисляем значения
функции в точках  –3, 0 и 3:

http://matematikalegko.ru/wp-content/uploads/2014/06/91.gif

Наименьшее
значение функции равно 0.
   Ответ: 0

Задача
7
. Найдите наименьшее значение функции у = х3 – 2х2 + х +3 на отрезке [1;4].

Решение. Найдём производную заданной функции:

http://matematikalegko.ru/wp-content/uploads/2014/06/102.gif

Найдем нули
производной, решаем квадратное уравнение:

2 – 4х + 1 = 0

Получим
корни:  х1 =
1    х1 =
1/3.   

Указанному в
условии интервалу принадлежит  только х = 1.

Найдём значения
функции в точках  1 и 4:

http://matematikalegko.ru/wp-content/uploads/2014/06/112.gif

Получили, что
наименьшее значение функции равно 3.

Ответ: 3

Задача
8.
Найдите наименьшее
значение функции у = х3 +
2 + х + 3 на отрезке
[– 4; –1].

Решение. Найдём производную заданной функции:

http://matematikalegko.ru/wp-content/uploads/2014/06/121.gif

Найдем нули
производной, решаем квадратное уравнение:

2 + 4х + 1 = 0

Получим корни:

http://matematikalegko.ru/wp-content/uploads/2014/06/131.gif

Указанному в
условии интервалу принадлежит  корень х = –1.

Находим значения
функции в точках  – 4, –1, –1/3 и 1:

http://matematikalegko.ru/wp-content/uploads/2014/06/141.gif

Получили, что
наименьшее значение функции равно – 28.

Ответ: – 28.

     
Рассмотрим способ определения наибольшего и наименьшего значения функций без
производной. Этот подход можно использовать, если с определением производной у
вас большие проблемы. Принцип простой – в функцию подставляем все целые
значения из интервала (дело в том, что во всех подобных прототипах ответом
является целое число).

Задача
9
. Найдите наименьшее значение функции у = 7 + 12хх3 на отрезке [–2;2].

Решение. Подставляем точки от  –2  до  2:

у(–2)=7+12 (–2) –
(–2)3 = – 9

у(–1)=7+12 (–1) –
(–1)3 = – 6

у(0)=7+12∙0 – 03 = 7

у(1)=7+12∙1 – 13 = 18

у(2)=7+12∙2 – 23 = 23

Наименьшее
значение равно –9.

Ответ: –9

Задача
10.
Найдите наименьшее
значение функции     у = х3 -27х   на отрезке [0;4].

Решение.

http://matematikalegko.ru/wp-content/uploads/2012/07/77421.gif

http://matematikalegko.ru/wp-content/uploads/2012/07/77421.gif

Задача
11.
Найдите наименьшее значение функции у = (х
3)2(х – 6) – 1  на отрезке  [4;6].

Решение. Так как интервал дан (при чём он небольшой), то здесь рекомендую
подставить целые значения из него  (4, 5 и 6) в функцию:

http://matematikalegko.ru/wp-content/uploads/2012/12/76.gif

Наименьшее
значение функции на заданном отрезке равно  – 5.
 

Ответ:  – 5

Задача 12. Найдите наибольшее
значение функции  у = (х + 6)2(х – 1) – 6
 

на отрезке [– 9;–2].

Решение. Данную задачу можно решать любым из двух способов. Интервал здесь
маленьким не назовёшь, но и в то же время он невелик.
  Решим её подстановкой всех
значений из интервала.

Подставим – 9, –
8, – 7, – 6, – 5, – 4, – 3, –2.

http://matematikalegko.ru/wp-content/uploads/2012/12/83.gif

Наибольшее
значение функции равно  – 6.
 

Ответ:  – 6

Функции, их исследование.

     Рассмотрим  
задачи, входящие в состав типовых заданий экзамена по  математике, связанные
с исследованием функций (где присутствует число е),
логарифмические,
показательные, иррациональные и другие.  
Для того чтобы решать задачи на нахождение наибольшего или
наименьшего значения, задачи на нахождение экстремумов, что производные элементарных функций нужно
знать наизусть, в теме производной без этого никак нельзя. Также необходимо
понимание того, что такое сложная функция, в указанной важно понимать свойства
производной для исследования функций.

Исследование функций с числом «е»
на наибольшее и наименьшее значение.

Решаем такие
задания следующим образом:

1. Находим производную.

2. Находим нули производной.

3. Определяем, какие из них принадлежат данному интервалу.

4. Вычисляем значения функции на границах интервала и точках п.3.

5. Делаем вывод (отвечаем на поставленный вопрос).

Показательные
функции.

Задача 13. Найдите  наименьшее
значение функции  ( х – 2)2 ех-2  на отрезке [1 ; 4]

Решение.  Сначала найдем производную данной функции, применив
правило для вычисления производной  произведения двух функций:

Производная обращается в нуль при х=0; х=2.

Данному промежутку принадлежит единственная точка х=2.

При переходе через эту точку производная функции
меняет знак  с минуса на плюс, эта точка является  единственной
точкой минимума на данном отрезке и  наименьшего значения на этом отрезке
функция достигает именно в этой

точке. Найдем наименьшее значение:

Ответ: 0

Задача 14. Найдите наименьшее значение функции у = (х–17)
ех–16   на отрезке [15;17].

Решение. Мы знаем, что для того, чтобы найти наибольшее или наименьшее
значение функции на отрезке, необходимо вычислить её значение на границах
заданного интервала и в точках, где производная равна нулю. Действуем по
алгоритму:

1. Найдём производную заданной
функции:

Исследование функкции

2. Найдем нули производной на
заданном отрезке, то есть приравниваем производную к нулю и вычислим корни
уравнения:

http://matematikalegko.ru/wp-content/uploads/2012/12/215.gif

Выражение ех-16 не
равно нулю ни при каких х, так как известно, что показательная функция имеет
положительные значения на всей области определения.

3. Определяем,
принадлежит ли найденная точка интервалу.

Точка х = 16 
принадлежит интервалу  [15;17]. Значит, значение функции будем вычислять в
точках  15, 16 и 17:

http://matematikalegko.ru/wp-content/uploads/2012/12/312.gif

*Учтите, что число е
≈ 2,71.  Это нецелое число и неконечная десятичная дробь, поэтому любое
выражение с этим числом в подобных задачах на ЕГЭ не является верным ответом,
но вы всё равно его проанализируйте. В данной задаче, если мы  –2 разделим
на число 2,71  то результат будет лежать в пределах от –1 до 0 (можно посчитать
столбиком для проверки).

4. Делаем вывод.

Таким образом,
наименьшее значение функции равно  –1.

Ответ: –1

    Ещё один путь
решения (без нахождения производной). Мы знаем, что ответом в задачах
на ЕГЭ в части  первой части,  должно быть, целое число, либо конечная
десятичная дробь.

Сразу подставляем в
функцию все целые значения из интервала (их всего три 15, 16 и 17), вычисляем и
выбираем наименьшее значение:

http://matematikalegko.ru/wp-content/uploads/2012/12/57.gif  

Ответ: –1

Задача 15. Найдите наибольшее значение функции у =
(22 – х)ех–21  
  на отрезке [16;25].

Решение. Найдём производную заданной функции:

http://matematikalegko.ru/wp-content/uploads/2012/12/142.gif

Найдем нули производной:

http://matematikalegko.ru/wp-content/uploads/2012/12/152.gif

Число ех-21 не
может быть равно нулю, так как степень положительного числа всегда даст в
результате число положительное, значит х = 21.

Полученное значение принадлежит
интервалу [16;25].

Вычислим значения
данной в условии функции в точках 16, 21  и 25:

*То есть на границах
интервала и в точке, где производная обращается в нуль.

http://matematikalegko.ru/wp-content/uploads/2012/12/162.gif

Первый результат
меньше единицы (это понятно и без вычислений).

Третий результат так
же меньше единицы (отрицательное число).

Значит наибольшее
значение функции на заданном  интервале равно 1.

*Помните, что ответы
с числом е (по требованиям ЕГЭ) не являются верными.

Ответ: 1

Задача 16. Найдите наибольшее
значение функции у = (2х2 – 10х + 10)ех

на
отрезке [–4; 3].

Решение. Необходимо определить значения на границах интервала, и в
точках, где производная обращается в нуль.

Найдём производную
заданной функции:

http://matematikalegko.ru/wp-content/uploads/2012/12/172.gif

Найдем нули производной:

http://matematikalegko.ru/wp-content/uploads/2012/12/182.gif

Произведение
множителей равно нулю, когда какой либо из этих множителей  равен нулю.

Число ех не может быть равно нулю, так
как степень положительного числа всегда даст в результате число положительное.

Значит  решением  являются
корни: х1=0  и  х2=3

Обе точки принадлежат интервалу [–4;3],
х=3 совпадает с границей интервала.

Вычисляем значения функции в точках:
– 4, 0 и 3:  

http://matematikalegko.ru/wp-content/uploads/2012/12/192.gif

Значит наибольшее
значение функции равно 10.
    Ответ: 10

*Как вы уже поняли,
можно в заданную функцию можно подставить все целые значения х из интервала, и
таким образом найти наибольшее значение функции. Но в данном случае придётся
перебрать 8 чисел (–4;–3;–2;–1;0;1;2;3).

Задача 17. Найдите наименьшее значение функции  у = (х +
44)2 е – 44 – х    на
отрезке  [– 46; –43] 

Решение. Найдём производную заданной функции:

http://matematikalegko.ru/wp-content/uploads/2012/12/202.gif

Обратите внимание, что результат мы
представили сразу в виде множителей, это будет удобно при вычислении нулей
производной.

Найдем нули производной:

http://matematikalegko.ru/wp-content/uploads/2012/12/216.gif

Решением являются
корни:  х1= – 44   и   х2= – 42. 

Заданному интервалу
[– 46;–43]  принадлежит только точка  х = – 44.

Вычисляем значения
функции в точках – 46, – 44 и – 43, то есть на границах интервала и в точке,
где производная равна нулю:

http://matematikalegko.ru/wp-content/uploads/2012/12/221.gif

Наименьшее значение
функции равно 0.
      Ответ:
0

*Как это задание
решить  можно  быстро?

Учитывая, что
ответом должно быть целое число, видно что значение данной функции будет целым
только при х= – 44 и х= 44.

указанному в условии
интервалу принадлежит х= – 44, вычисляем:

http://matematikalegko.ru/wp-content/uploads/2012/12/231.gif

Задача  18.  Найти
наибольшее и наименьшее значения функции

f (x) = 5 на отрезке
[4; 40].

Решение. Находим критические точки
функции, лежащие внутри данного отрезка:

Вычисляем
значения функции на концах отрезка и в критической точке:
f (4) = 11, f (12) = 13, f (40) = 5. Из полученных значений
выбираем наибольшее и наименьшее:

,       .

Ответ: , .

Задача  19. Найти наибольшее значение
функции
f (x) = x ln 5 – x ln x на отрезке .

Решение.  = ln 5
ln x – 1 =   при .
Сравнение значений функции на концах отрезка и в критической точке приводит к
сложным вычислениям. Вместо этого проведем исследование функции на
монотонность. Учитывая непрерывность функции в точке  и тот факт, что при  производная положительна, а при  отрицательна, приходим к выводу, что на
промежутке  функция возрастает, а на
промежутке  убывает. Это и означает, что
значение функции в точке  является
наибольшим из всех значений функции на данном отрезке.

Задача 20. Найти наименьшее значение
функции

y = (x  1) (x  2) (x  3) (x  4) + 3.

Решение. Преобразуем функцию
следующим образом:

y = (x2 – 5 x + 4) (x2 – 5 x + 6) + 3
=

= (x2 – 5 x + 4)2 +
2 (x2 – 5 x + 4) + 1 + 2 = (x2 – 5 x + 5)2 + 2.

Ясно, что
наименьшее значение функции равно 2 и достигается оно при .

Ответ: 2.

Задача 21.  Найдите
наибольшее значение функции .

Решение. Данная функция представляет собой выражение,
числитель которой – положительное число, а знаменатель – квадратный трехчлен с
отрицательным дискриминантом.

Следовательно, наибольшее значение данной функции
достигается при наименьшем значении знаменателя при .
Соответственно, .

Ответ: 2.

Задача  22. Найти наибольшее значение
функции
f (x) = x ln 5 – x ln x на отрезке .

Решение.  = ln 5 – ln x – 1 =   при .
Сравнение значений функции на концах отрезка и в критической точке приводит к
сложным вычислениям. Вместо этого проведем исследование функции на
монотонность. Учитывая непрерывность функции в точке  и тот факт, что при  производная положительна, а при  отрицательна, приходим к выводу, что на
промежутке  функция возрастает, а на
промежутке  убывает. Это и означает, что
значение функции в точке  является
наибольшим из всех значений функции на данном отрезке.

Вычислить
наибольшее ( наименьшее) значение.

Задачи с логарифмами

Задача 23. Найдите
наименьшее значение функции у = 5х – ln (х+5)5  на отрезке
[– 4,5;0].

Необходимо вычислить значение функции на концах интервала, и в
точках экстремума, если таковые имеются на данном интервале, и выбрать
наименьшее из них.

Вычисляем
производную, приравниваем её к нулю, решаем уравнение.

Найдем производную
заданной функции:

Нахождение наибольшего или наименьшего значения на отрезке.

Найдем нули
производной на заданном отрезке:

http://matematikalegko.ru/wp-content/uploads/2013/04/214.gif

*Дробь равна нулю
тогда, когда числитель равен нулю.

Точка  х= –
4  принадлежит заданному интервалу. 

Таким образом,
вычисляем значение функции в точках: – 4,5; – 4; 0.

http://matematikalegko.ru/wp-content/uploads/2013/04/33.gif

Значения с
логарифмами, которые мы получили, вычислить (или проанализировать) можно. И вы
убедитесь, что наименьшим значением функции на данном отрезке  является
   “– 20”.

Но вычислять их не
обязательно. Почему? Мы знаем, что ответом должно быть либо, целое число, либо
конечная десятичная дробь (это условие ЕГЭ в части В). А значения с
логарифмами:  – 22,5 – ln 0,55  и   – ln3125
 такого ответа не дадут.

Кроме того, убедится
в том, что в точке х=–4 функция приобретает минимальное значение, можно
определив знаки производной на интервалах от (– 5:– 4) и (– 4;+∞). 

Теперь информация
для тех, у кого с производной и пониманием того, как решать подобные задачи,
нет трудностей. Как можно обойтись без вычисления производной и без лишних
расчётов?

Итак, если учесть,
что ответом должно быть целое число, либо конечная десятичная дробь, то такое
значение мы можем получить только тогда, когда х будет являться целым числом,
либо целым с конечной десятичной дробью и при этом под знаком логарифма в
скобках у нас будет единица или число е. В противном случае, мы не сможем
получить оговоренное значение. А это возможно только при  х = – 4.

Значит, в этой точке
значение функции будет наименьшим, вычислим его:

http://matematikalegko.ru/wp-content/uploads/2013/04/44.gif

Ответ: – 20.

Задача 24. Найдите наименьшее
значение функции у = е – 8е х + 9 на
отрезке [0; 2].

Решение. Необходимо определить значения на границах интервала, и в
точках, где производная обращается в нуль.

Найдём производную заданной
функции:  у
׳ =  (е – 8е х + 9)׳
=
2 е 2 х  – 8 е х  

Найдем нули производной:

у ׳ = 0 ;      – 8 е х  =
0;  
  2 е х
(  е х   – 4 ) = 0.

е х   0 тогда    е х
  – 4 = 0 ; 
 е х   =
4 ;
 х = ln 4.    ln 4[0; 2].

Произведение
множителей равно нулю, когда какой либо из этих множителей  равен нулю.

Число ех не может быть равно нулю, так
как степень положительного числа всегда даст в результате число положительное.

Значит  решением  являются
корнь:
х = ln 4

Эта  точка  принадлежат интервалу
[0;2],

Вычисляем значения функции в точках: 
0 ,
ln 4  и 2.   

у(
0)  = е 2·0 – 8е 0 + 9 = 1 – 8 + 9 =2

у(2)
 = е 2·2 – 8е 2 + 9 =  е 4 – 8е2
+ 9

у
(
ln 4)   = е 2ln 4 – 8е ln 4 + 9 = е ln 16 – 8е ln 4 + 9 = 16 – 32 +9
= -7.

Значит наименьшее
значение функции равно – 7.
    Ответ: – 7.

Функции   заданные 
вида   
у = е – 8е х + 9 можно решить
 только с помощью производной. При решении  получается корень  выраженный
через натуральный лагорифм.

Тригонометрические функции

Основная сложность тригонометрических
функций состоит в том, что при решении уравнений возникает
бесконечное множество корней. Например, уравнение sin x = 0
имеет корни x = πn, где n 
 Z. Ну и как
отмечать их на координатной прямой, если таких чисел бесконечно
много?

Ответ прост: надо
подставлять конкретные значения n. Ведь в задачах B12
с тригонометрическими функциями всегда есть ограничение —
отрезок [a; b]. Поэтому для начала берем n = 0,
а затем увеличиваем n до тех пор, пока соответствующий
корень не «вылетит» за пределы отрезка [a; b]. Аналогично,
уменьшая n, очень скоро получим корень, который меньше нижней границы.

Несложно
показать, что никаких корней, кроме полученных в рассмотренном процессе,
на отрезке [a; b] не существует. Рассмотрим теперь этот
процесс на конкретных примерах.

Задача1. Найти наибольшее значение функции
y = 4tg x − 4x + π − 5
на отрезке [−π/4; π/4].

Решение. Вычисляем производную:
y’ = (4tg x − 4x + π − 5)’ =
4/cos 2x − 4.

Затем решаем
уравнение: y’ = 0 

4/cos 2x − 4 = 0 
…  x = πn, n  Z.

Выделим из этой формулы корни,
подставляя конкретные n, начиная с n = 0:
n = 0 
x = 0. Этот корень нам подходит.
n = 1 
x = π. Но π > π/4, поэтому
корень x = π и значения n > 1
надо вычеркнуть.
n = −1 
x = −π. Но π < −π/4, поэтому
x = π и n < −1 тоже вычеркиваем.

Из всего многообразия корней остался лишь
один: x = 0. Поэтому вычисляем значение функции для
x = 0,x = π/4и x = −π/4.
y(0) =4tg 0 − 4·0 + π − 5 =π − 5;y(π/4) =4tg (π/4) − 4·π/4 + π − 5 =1;
y(π/4) =
4tg (−π/4) − 4·(−π/4) + π − 5 =
… = 2π − 9.

Теперь заметим, что
π = 3,14… < 4, поэтому
π − 5 < 4 − 5 < 0
и 2π − 9 < 8 − 9 < 0.
Получается одно положительное число и два отрицательных. Мы ищем
наибольшее — очевидно, это y = 1.     Ответ: 1

     Заметим, что в последней задаче
можно было и не сравнивать числа между собой. Ведь из чисел
π − 5, 1 и 2π − 9 в бланк ответов может быть
записана лишь единица. Действительно, как написать в бланке, скажем,
число π? А никак. Это важная особенность первой части
ЕГЭ по математике, которая значительно упрощает решение многих задач.
И работает она не только в B12.

Иногда при исследовании функции
возникают уравнения, у которых нет корней. В таком случае задача
становится еще проще, поскольку остается рассмотреть лишь
концы отрезка.

Задача2. Найти наименьшее значение функции
y = 7sin x − 8x + 5
на отрезке [−3π/2; 0].

Решение. Сначала находим производную: y’
= (7sin x − 8x + 5)’ = 7cos x − 8.

Попробуем решить
уравнение: y’ = 0 

7cos x − 8 = 0 
cos x = 8/7.
Но значения cos x всегда лежат на отрезке [−1; 1],
а 8/7 > 1. Поэтому корней нет.

Если корней нет,
то и вычеркивать ничего не надо. Переходим к последнему
шагу — вычисляем значение функции:
y(−3π/2) = 7sin (−3π/2) − 8·(−3π/2) + 5 =
… = 12π + 12;
y(0) = 7sin 0 − 8·0 + 5 = 5.

Поскольку
число 12π + 12 в бланк ответов не записать, остается
лишь y = 5.

Ответ: 5

Задача3.  Найдите
наименьшее значение функции y = 5cosx – 6x + 4

на отрезке [–3П/2; 0].

Найдём производную заданной функции:

http://matematikalegko.ru/wp-content/uploads/2013/11/213.gif

Найдем нули производной на заданном отрезке:

http://matematikalegko.ru/wp-content/uploads/2013/11/313.gif

Известно, что  – 1 ≤ sin x ≤  1, то есть уравнение не
имеет решения.

Это означает, что в пределах заданного интервала нет точек
минимума и максимума. Производная будет отрицательна при всех значениях
переменной. Почему?

Если учесть, что  – 1≤sinx≤ 1,  то получаем

– 1≤sinx≤1   =>  5
≥ –5sinx≥ –5   =>   –1 ≥ –5sinx–6 ≥ –11

то есть значение выражения (производной) «–5cosx – 6» лежит в
пределах от  – 11 до  – 1 включительно.

Следовательно на указанном интервале функция убывает, и наименьшее
значение будет в крайней правой  точке, то есть при х = 0. Таким образом,

http://matematikalegko.ru/wp-content/uploads/2013/11/46.gif

Ответ: 9

 Задача 4.
Найдите наименьшее значение функции y = 7sin x – 8x + 9

на отрезке [–3П/2; 0].

Найдём производную заданной функции:

http://matematikalegko.ru/wp-content/uploads/2013/11/52.gif

Найдем нули производной на заданном отрезке:

http://matematikalegko.ru/wp-content/uploads/2013/11/62.gif

Известно, что  – 1 ≤ cos x ≤  1, то есть уравнение не
имеет решения.

Это означает, что в пределах заданного интервала нет точек
минимума и максимума. Производная отрицательна при всех значениях переменной,
значение производной лежит в пределах от   – 15 до  – 1
включительно.

Значит на указанном интервале функция убывает. 

Следовательно наименьшее значение функции на заданном отрезке
будет в правой крайней точке, то есть при х = 0.

http://matematikalegko.ru/wp-content/uploads/2013/11/73.gif

Ответ: 9

Задача 5.  Найдите наибольшее значение функции

http://matematikalegko.ru/wp-content/uploads/2013/11/82.gif

Найдём производную заданной функции:

http://matematikalegko.ru/wp-content/uploads/2013/11/92.gif

Найдем нули производной на заданном отрезке:

http://matematikalegko.ru/wp-content/uploads/2013/11/102.gif

Точка x = П/6, принадлежит заданному интервалу.

Вычислим значение
функции в точках: 0, П/6, П/2.

http://matematikalegko.ru/wp-content/uploads/2013/11/113.gif

Если учесть, что число
Пи равно 3,14 а корень из трёх  ≈ 1,73  то значения  вычислить
будет не трудно:

http://matematikalegko.ru/wp-content/uploads/2013/11/122.gif

Значит наибольшим
значением функции на отрезке  будет 12. Данные приближённые значения можно
и не вычислять. Достаточно  помнить то, что ответом в задачах части В
является целое число, а там где присутствует неизвлекаемый в целых числах
корень, целое число мы никак не получим.

Ответ: 12

*Примечание. Корень
уравнения мы записали сразу с учётом данного в условии отрезка, поэтому период
косинуса в результате не записан.

Задача 6.  Найдите наибольшее
значение функции

http://matematikalegko.ru/wp-content/uploads/2013/11/132.gif

Найдем производную заданной функции:

http://matematikalegko.ru/wp-content/uploads/2013/11/142.gif

Найдем нули производной на заданном отрезке:

http://matematikalegko.ru/wp-content/uploads/2013/11/152.gif

Значит уравнение не имеет решения, так  как  – 1 ≤ cos x
≤  1.

Учитывая данное ограничение, производная  на данном отрезке
имеет отрицательное значение:

http://matematikalegko.ru/wp-content/uploads/2013/11/162.gif

Следовательно она убывает. 

Таким образом, наибольшее значение функции на заданном отрезке
будет в левой  крайней точке, то есть при х = – 5П/6.

http://matematikalegko.ru/wp-content/uploads/2013/11/172.gif     Ответ: 32

Задача 7.  Найдите наибольшее
значение функции у = 15х – 3
sin x + 5 на отрезке  .http://matematikalegko.ru/wp-content/uploads/2012/07/26695.gif

Задача
8.
  Найдите наименьшее значение функции y =  3 + – 5х – 5 cosx на отрезке
 .

http://matematikalegko.ru/wp-content/uploads/2012/07/266931.gif

Задача
9.
  Найдите наименьшее значение функции y =  3 – + 5х – 5 sinx на отрезке  .

http://matematikalegko.ru/wp-content/uploads/2012/07/774992.gif

2.        
Методические рекомендации обучения учащихся решению заданий вычислению
точек экстремума

Решение задач на
нахождение точек максимума и минимума (точек экстремума) функции
основывается на следующих утверждениях:

Признак
максимума
.
Если функция f непрерывна в точке  на интервале
 на интервале ,
то  – точка максимума функции f (упрощенная
формулировка: если в точке   производная 
меняет знак с плюса на минус, то  – точка
максимума).

      Признак минимума. Если функция f непрерывна в точке  на интервале  на
интервале ,  то – точка минимума функции f (упрощенная
формулировка: если в точке    производная 
меняет знак с минуса на плюс, то  – точка
минимума).

Может оказаться
очень полезным следующее свойство непрерывных функций: «Если функция  y = f(x)
имеет на промежутке I единственную точку экстремума x0  и  эта точка
является точкой минимума, то в ней  достигается наименьшее значение функции на
данном промежутке». Аналогичное утверждение справедливо для точки максимума и
наибольшего значения функции.

Например, если
функция y = f(x), непрерывная на отрезке [a;b], имеет на промежутке (a;b)
единственную точку экстремума xо  и эта точка является точкой
максимума функции, то наибольшее значение функции на отрезке [a;b] равно f(xо).

Иногда при
решении задач на исследование функций оказывается, что на данном промежутке
точек экстремума нет. Такой ситуации не надо пугаться: она означает, что на
этом промежутке производная принимает значения одного знака, т. е. функция
является монотонной на нем. Остается заметить, что если функция возрастает на
отрезке, то наибольшее значение на нем достигается в правом конце отрезка, а
наименьшее — в левом; если функция убывает на отрезке, то наибольшее значение
на нем достигается в левом конце отрезка, а наименьшее — в правом.

Практические
правила исследования функции на максимум и минимум с помощью производной.

1)  Найти производную функции

2)  Найти критические точки
функции ,т.е. точки, в которых обращается в нуль или терпит разрыв.

3)  Исследовать знак производной в промежутках, на которые найденные
критические точки делят область определения функции .
При этом критическая точка есть точка
минимума (максимума), если производная меняет знак при переходе через . Если же в соседних промежутках,
разделённых критической точкой , знак 
производной не меняется, то в точке функция не
имеет ни максимума, ни минимума.

4)  Вычислить значения функции в
точках максимума и минимума.

     Рассмотрим примеры:

Пример 1.Найдите точку
максимума функции  у = х3 – 48х +17 .

Решение. Найдем производную
заданной функции: у’ = 3х2 – 48 = 3(х2 – 16) = = 3(х – 4)
(х + 4).

Найдем нули
производной:   3 (х – 4) (х + 4) =0
  х =4;  х= – 4

Определим знаки
производной функции и изобразим на рисунке поведение функции:

http://reshuege.ru/get_file?id=1005

Искомая точка максимума 
х = – 4.

Ответ: -4

Пример 2. Найдите точку минимума
функции у = (х + 18)ех-18   

Решение.  1. Найдём производную заданной функции:

у’ = (х +18)’ ех-18   + (х + 18) (ех-18
)’ = ех-18   + (х + 18) ех-18 = (х + 19)
ех-18

2. Найдем нули
производной:

у’ = 0   (х + 19) ех-18
= 0

Получаем, что х =
–19.  

Выражение ех-18 не
равно нулю ни при каких х, так как известно, что показательная функция имеет
положительные значения на всей области определения.

3. Определим знаки
производной функции на интервалах (подставляем любые произвольные значения в
производную) и изобразим на рисунке поведение функции:

у’ (-20) = (-20 + 19) е -20 – 18 = -1 е  – 38
     

у’ (-10) = (-10 + 19) е -10 – 18 = 9 е  – 28
     

http://matematikalegko.ru/wp-content/uploads/2012/12/10.jpg

В точке х = –19 функция
меняет знак с отрицательного на положительный,  значит это искомая точка
минимума.
       

Ответ: –19 

Как решать быстрее
данный тип задач?

Когда мы получили
производную и приравняли её к нулю:

(х +
19) е х–18
 = 0

Далее получили, что х = –19. Данное
решение и будет являться ответом задачи т.к. е х–18
  0

Пример 3.  Найдите
точку максимума функции у = (3х2 – 15х + 15) е7–х.

Решение. Найдём производную заданной функции:

у’ = (3х2 –
15х + 15)’ е7–х +  (3х2 – 15х + 15) (е7–х)’
= (6х – 15) е7–х +  (3х2 – 15х + + 15) е7–х 
( 7 – х)’ = (6х – 15) е7–х  +  (3х2 – 15х + 15)
е7–х  (–1) = (6х – 15) е7–х   – – (3х2 –
15х + 15) е7–х  = ( (6х – 15)  –  (3х2 – 15х +
15) ) е7–х  = (6х – 15 –  3х2 + + 15х – 15) е7–х 
= (– 3х2 + 21х – 30) е7–х  = –3 (х –5) (х – 2) е7–х  
= 0

Найдем нули производной:

у’ =0  -3 (х -5) (х –
2) е7– х   = 0

-3
(х -5) (х – 2) =0 либо е7 – х   = 0

Число е7-х не
может быть равно нулю, так как степень положительного числа всегда даст в
результате число положительное. 

Решаем  – 3
(х–5)(х–2) = 0.  Получим х1
 = 5  и  х2 = 2 .

Определим знаки
производной функции (подставляя любые значения из интервалов в найденную
производную) и изобразим на рисунке поведение функции:

у'(0) = -3 (0 – 5) (0 –
2) е7– 0 = – 30 е7
0

у'(3) = -3 (3 – 5) (3 –
2) е7– 3 = 6 е4
0

у'(6) = -3 (6 – 5) (6 –
2) е7– 6 = – 12 е
0

102

В точке х = 5 функция меняет знак с
положительного на отрицательный, значит это искомая точка максимума.
   

 Ответ: 5.

Пример 4. Найдите точку максимума функции у = х2 – 34х
+ 140lnх–10.

Решение. По свойству логарифма выражение, стоящее под его знаком больше
нуля, то есть х > 0.

Функцию будем
рассматривать на интервале (0; +∞).

Найдём производную
заданной функции:

(х2
34х +140
ln x -10)’ = 2x -34 +

Найдем нули
производной:

у’ =0  2x -34 +   = 0

2
34х + 140 = 0

х2
17х + 70 = 0

Решая квадратное
уравнение, получим: D = 9    х1 = 10   х2 = 7.

Точка х = 0 
не входит в область определения функции и в ней производная не существует.
Отмечаем на числовой оси три точки 0, 7 и 10.

Ось ох разбивается
на интервалы:  (0;7),  (7;10), (10; +∞).

Определим знаки
производной функции, подставляя произвольные значения из полученных интервалов
в найденную производную, и изобразим на рисунке поведение функции:

у'(5) =2 ·5 -34 +    0

у'(8) =2 ·8 -34 +    0

у'(14) =2 ·14 -34 +    0

http://matematikalegko.ru/wp-content/uploads/2013/04/93.gif

Таким образом, в точке х = 7 производная функции меняет знак с
положительного на  отрицательный,  значит это искомая точка
максимума.
      Ответ:
7

Пример 5.. Найдите точку максимума
функции 
.

Решение. Вычислим
производную функции, используем формулу производной показательной и производной
сложной функции:

13

Найдем нули производной. Известно, что произведение равно нулю
тогда и только тогда, когда хотя бы один из множителей равен нулю, а другие при
этом не теряют смысла. В данном случае:

–30 +
12х – х2 =
0     или     –2х + 12 = 0

Решим первое:   –30 + 12х – х2 = 0

D= 122 – 4 ·(
-1) ·(- 30) = 144 -120 = 24.

х1 =
 = =  = 6 –  ;    х2
=
 = =  = 6 +

Второе уравнение:             –2х + 12 = 0;      х = 6

У нас получилось
три точки, в которых  производная функции равна нулю. Если записать
приближенно (первые две), то получим:  3,56; 6 и  8,44.

Определим знаки производной на интервалах, которые получаются
разбие-нием числовой оси указанными выше точками. Возьмём точки 0, 5, 7 и 9:

у'(0) = ( – 30 + 12 ·0 – 02) · ln8·(-2·0  +12) = -30 ln8·12 0  функция убывает.

у'(5) = ( – 30 + 12 ·5 – 52) · ln8·(-2·5  +12) = 5 ln8·2 0  функция возрастает.

у'(7) = ( – 30 + 12 ·7 –72) · ln8·(-2·7  +12) = 5 ln8· (-2) 0  функция убывает.

у'(9) = ( – 30 + 12 ·9 –92) · ln8·(-2·9  +12) = -3 ln8· (-2) 0  функция возрастает .

15

Получили, что в точке х = 6 производная функции меняет свой знак с
положительного на отрицательный, а это означает, что данная точка есть точка
максимума функции.

Ответ: 6

        Комментарий:
в данном случае область определения не имеет ограничений, то есть при всех х
функция имеет значение. При вычислении нулей функции получили три точки. Две из
них иррациональны (имеется корень). На основании этого мы уже могли бы сделать
вывод о том, что точка х = 6 это точка максимума, так ответом должно быть целое
число или конечная десятичная дробь. Но, всё же, рекомендую определить знаки
производной на интервалах, чтобы убедиться какая точка является точкой
максимума. В случае, если корни квадратного трёхчлена будут целыми, то это
нужно сделать обязательно.

Если будет стоять
вопрос о нахождении максимального  (минимального) значения функции на
определённом интервале, то тут действуйте по стандартному алгоритму (посмотрите
задания на сайте, их достаточно).

В любом случае,
свойства производной для исследования функции, табличные значения производных и
алгоритм нахождения точек максимума (минимума) нужно знать обязательно.

Исследования степенной функции на максимум
и минимум.

Пример 6. Найдите точку максимума функции у = (х – 2)2
– 4) + 5. 

Решение. Для этого найдём производную, приравняем её к нулю, найдём корни
полученного уравнения:

у’ = ((х 2)24) + 5)’
= ((х

2)24))’  = ((х 2)2
)
4) +(х 2)2 4)’ =

= 2(х 2) (х 4) + (х 2)2  = (х 2) (2х
8 + х  2) = (х 2) (3х 10).

Находим нули
производной:

у’=0 значит (х -2) (3х-10)= 0  

х-2=0,  
х1 = 2  или 3х-10 =0 ,   х2 =

Полученные точки
разбивают числовую ость на интервалы.

Посмотрим, как
ведёт себя график функции на этих интервалах. Возьмём значения из них,
подставим их в производную и определим знак:

у’ (0) = (0 -2) (3·0-10) = 20

у’ (3) = (3 -2) (3·3-10) = -1

у’ (4) = (4 -2) (3·4-10) = 4

http://matematikalegko.ru/wp-content/uploads/2012/12/315.gif

На интервале
(–∞;2) функция возрастает, на интервале (2;10/3) функция убывает.  Значит
 х = 2  это точка максимума.

Ответ: х = 2.

Пример 7.Найдите точку минимума функции у = х3+5х2+7х–5.

Решение. Найдём
производную функции:

http://matematikalegko.ru/wp-content/uploads/2014/06/4.gif

Найдем нули производной:  3х2 + 10х + 7 = 0Решая квадратное уравнение получим:     
http://matematikalegko.ru/wp-content/uploads/2014/06/5.gif

Определяем знаки производной функции на интервалах и отметим их на
эскизе. Подставляем произвольное  значение из каждого интервала в
выражение производной:

у(–3)’ = 3∙(–3)2 + 10∙(–3) + 7 = 4 > 0

у(–2)’= 3∙(–2)2 
+ 10∙(–2) + 7 = –1 < 0

у(0)’= 3∙02 – 10∙0 + 7 = 7 > 0

http://matematikalegko.ru/wp-content/uploads/2014/06/6.gif

В точке х = –1 производная меняет свой знак с отрицательного на
положительный, значит это есть искомая точка минимума.

Ответ: –1

Пример 8. Найдите точку максимума
функции у = 7+12х–х3

Решение. Найдём
производную функции: у’ = (7+12х–х3)= 12 – 3х2

Найдем нули производной:

12 –
2 = 0

х2 = 4х1 = 2; х2
= 2

Это точки возможного максимума (минимума) функции.

Определяем знаки производной функции на интервалах и отметим их на
эскизе. Подставляем произвольное значение из каждого интервала в выражение
производной:

у(–3)’=12
– 3∙(–3)2 = –15 < 0

у(0)’=12
– 3∙02 = 12 > 0

у(3)’=12
– 3∙32 = –15 < 0

http://matematikalegko.ru/wp-content/uploads/2014/06/9.gif

В точке х = 2 производная меняет свой знак с положительного на отрицательный,
значит это есть искомая точка максимума.

Ответ: 2.

Для этой же функции точкой минимума является точка х = – 2.

Пример 9. Найдите точку максимума
функции у =  9х2– х3.

Решение. Найдём
производную функции:  у’ =  (9х2– х3)’= 18х – 3х2

Найдем нули производной:

18х
–3х2 = 0

3х(6 –
х) = 0
 х1 = 0; х2= 6.

Определяем знаки производной функции на интервалах и отметим их на
эскизе. Подставляем произвольное значение из каждого интервала в выражение
производной:

у(–1)’=18
(–1) –3 (–1)2 =
–21< 0

у(1)’=18∙1
–3∙12 = 15 > 0

у(7)’=18∙7
–3∙72 = –1< 0

http://matematikalegko.ru/wp-content/uploads/2014/06/12.gif

В точке х = 6 производная меняет свой знак с положительного на
отрицательный, значит это есть искомая точка максимума.
 

Ответ: 6

Для этой же функции точкой минимума является точка х = 0.

Иррациональные функции

Пример 10. Найдите
точку минимума  функции  у =
 – 2х +1

Решение: . Производная
обращается в ноль при х = 4. В точке х = 4 производная меняет знак с минуса на
плюс. Эта точка является единственной точкой минимума.

Ответ: 4

Пример 11. Найдите точку максимума
функции
.

Решение. Сначала
определим, при каких х функция имеет смысл (найдём область определения
функции). Так как подкоренное выражение есть число неотрицательное, то решаем
неравенство:

13 +
6х – х2 ≥ 0

2

Данные корни разбивают ось х на три интервала.

Проверим при каких значениях х неравенство будет верным. Подставим
из каждого интервала любое значение х в неравенство:

22

Значит решением неравенства будут являться все значения х
принадлежащие интервалу (включая границы):

3*Приближенно полученные
выражения равны:   

4

Область определения данной функции найдена.

Вычислим производную функции. Это сложная функция:

5

Найдем нули производной:

http://matematikalegko.ru/wp-content/uploads/2014/05/73.gif

Дробь равна нулю тогда, когда её числитель равен нулю, значит:

6 –
2х  = 0

х = 3

Полученное 
значение х входит в область определения и разбивает её на два отрезка.
Определим знаки производной на каждом из них (подставим выборочно любые
значения в выражение производной), например 2 и 4:

http://matematikalegko.ru/wp-content/uploads/2014/05/83.gif

Получили, что в точке х = 3 производная функции меняет свой знак с
положительного на отрицательный, а это означает, что данная точка есть точка
максимума.

Ответ: 3

      Комментарий:
представленное решение – это полное, математически грамотное решение, то есть
такое как должно быть.  Дело в том, что для составления «полной картины», в
первую очередь необходимо найти область определения. Дело в том, можно сразу
находить  производную, затем её «нули» и далее установить имеет ли функция
значение при этом х. Затем определить знаки в «соседних» точках и станет
понятно является ли эта точка точкой максимума (или минимума). Да, можно и так.

Пример 12. Найдите точку
максимума функции  у =
ln(х+5)5 – 5х

Решение. Найдём производную данной функции .

Производная меняет знак в единственной точке х= 4,причём знак производной в этой точке меняется 
с плюса на минус.  Следовательно, эта точка является единственной точкой
максимума данной функции.

Пример 13. Найдите точку максимума
функции у =log7(–2 – 12х – х2) + 10. 

Решение. Вычислим
производную функции, используем формулу производной логарифма и производной
сложной функции:

8

Найдем нули производной:

http://matematikalegko.ru/wp-content/uploads/2014/05/104.gif

Дробь равна нулю тогда и только тогда, когда её числитель равен
нулю:

– 2х
–12 = 0

х = –
6

Данное значение обращает подлогарифмическое выражение в
положительное:

–2 –
12∙(–6) – (–6)2 = 34

то есть оно принадлежит области определения функции.

Определим знаки производной в «соседних» точках, например возьмем
точки  –7 и –5:

11

Получили, что в точке х = – 6 производная функции меняет свой знак
с положительного на отрицательный, а это означает, что данная точка есть точка
максимума функции.

Ответ: – 6

      Комментарий: здесь мы не стали находить область определения.
Сразу вычислили производную и нашли х, при котором производная обращается в
нуль. Затем определили знаки производной на интервалах полученных разбиением
числовой оси точкой х =  – 6 и сделали вывод.

Если в подобном задании (с логарифмом) будет стоять вопрос о
вычислении наибольшего (наименьшего) значения функции, то также вычисляйте его
в точке х, полученной при решении f′(х) = 0.

Пример 14.  Найдите точку максимума
функции y = (2x –3) cos x – 2sin x + 5 

принадлежащую
промежутку (0;
/2).

Решение. Найдём производную функции:

у‘ = ((2x –3) cos x – 2sin
x + 5 )’ = ((2x –3) cos x)’ – (2sin x)’ + 5’=
(2x –3)’ cos x + +(2x–3)(cos x)’ – (2sin x)’
+ 5′ = 2 cos x + (-sin x)(2
х 3) 2cosx = sin x (2х3)

Решаем уравнение:

sin x (2х 3) = 0

Произведение равно
нулю тогда и только тогда, когда хотя бы один из множителей равен нулю, и
другие при этом не теряют смысла. Следовательно:

sin x =0  или   2х 3 = 0

Решаем уравнение
     – sin x = 0:  х =

В условии дан
промежуток (0;
/2). Ему не принадлежит ни один из полученных корней. *Обратите
внимание, что указанные границы исключены (скобки круглые).

Решаем уравнение:
2х – 3 = 0, получим х = 1,5.

Запишем данный
промежуток в радианах, получим: (0;1,57), так как

http://matematikalegko.ru/wp-content/uploads/2013/10/54.gif

Следовательно,
полученное значение принадлежит промежутку (0;
/2): 0

Конечно, нам
интуитивно понятно, что полученная точка это и есть точка максимума, и,
казалось бы, в дальнейших вычислениях и рассуждениях нет необходимости. Но
любая задача данного типа должна быть решена до конца по указанному алгоритму.
Это важно!

Полученное
значение х разбивает данный промежуток на два других. Определим знаки
производной функции, подставляя произвольные значения из полученных промежутков
(0;1,5) и (1,5;1,57) в найденную производную, и изобразим на рисунке поведение
функции:

http://matematikalegko.ru/wp-content/uploads/2013/10/74.gif

В подобных случаях
необязательно вычислять значения выражений. Важно установить их знаки
(положительный либо отрицательный). Например, мы видим, что выражение:

(3,14/2)
– 3    имеет отрицательный знак

3,14 –
3    имеет положительный знак

 В целом
этого достаточно для определения знака выражения.

http://matematikalegko.ru/wp-content/uploads/2013/10/84.gif

Таким образом, в
точке х = 1,5 функция меняет знак с положительного на отрицательный.  Это
означает, что данная точка является точкой максимума функции на заданном
промежутке.

Ответ: 1,5  

Пример 15. Найдите точку минимума функции
y = x3/2
 – 3x + 1

Решение. Найдём производную заданной функции:

Найдите точку минимума функции y = x^(3/2) – 3x + 1

Найдем нули производной:

http://matematikalegko.ru/wp-content/uploads/2014/07/101.gif

Решаем уравнение:

http://matematikalegko.ru/wp-content/uploads/2014/07/111.gif

Определим знаки производной функции и
изобразим на рисунке поведение функции. Для этого подставим произвольные
значения из полученных интервалов в производную:

http://matematikalegko.ru/wp-content/uploads/2014/07/121.gif

http://matematikalegko.ru/wp-content/uploads/2014/07/121.gif

В точке х = 4, производная меняет
знак с отрицательного на положительный, это означает, что данная точка является
точкой минимума.

Ответ: 4

http://matematikalegko.ru/wp-content/uploads/2014/07/171.gif 

http://matematikalegko.ru/wp-content/uploads/2014/07/171.gif

В точке х = 4, производная меняет
знак с положительного на отрицательный, это означает, что данная точка является
точкой максимума.

Ответ: 4

Пример 16. Найдите точку минимума функции у= 4х– ln (х+5)+8

Решение. Сразу
запишем, что х+5>0 (по свойству логарифма), то есть х>–5.

Рассматривать
функцию будем на интервале  (– 5;+∞).

Найдём производную
заданной функции:

у’= (4х– ln
(х+5)+8)’ = 4

Найдем нули
производной:

у’=0   4 –  = 0;  х = – 4 ,75

Точка х
= –5  не входит в область определения функции и в ней производная не
существует. Отмечаем на числовой оси две точки –5 и –4,75. Определим
знаки производной функции, подставляя произвольные значения из интервалов
(–5;–4,75) и (–4,75;+∞) в найденную производную, и изобразим на рисунке
поведение функции:

у’ (- 4,9)  =  4 –
 ;          у’ (- 4)  =  4 –  ;         

http://matematikalegko.ru/wp-content/uploads/2013/04/64.gif

Таким образом, в точке х= –4,75  производная функции меняет
знак с отрицательного на положительный,  значит это искомая точка
минимума.
    Ответ:
– 4,75   

 Пример 17.
Найдите точку минимум функции у =

Решение. Найдем производную данной функции:

Определим промежутки знакопостоянства производной, приведя
полученное выражение к общему знаменателю и  разложив числитель на множители:

В точке х = 5 производная меняет знак с минуса на
плюс, следовательно, эта точка и является единственной точкой минимума.

Ответ: 5.

 Пример18. Найти точку максимума функции
y = sin x − 5x·sin x − 5cos x + 1,
принадлежащую отрезку [−π/3; π/3].

Решение. Вычисляем производную: y’ = (sin x − 5x·sin x − 5cos x + 1)’ =
… = cos x − 5x·cos x =
(1 − 5x)·cos x.

Затем решаем
уравнение: y’ = 0 

(1 − 5x)·cos x = 0 
…  x = 0,2
или x = π/2 + πn, n 
 Z.

С корнем x = 0,2 все понятно,
а вот формула x = π/2 + πn требует дополнительной
обработки. Будем подставлять разные значения n, начиная с n =
 0.

n = 0  x = π/2.
Но π/2 > π/3, поэтому корень x = π/2
не входит в исходный отрезок. Кроме того,
чем больше n, тем больше x, поэтому нет смысла
рассматривать n > 0.

n = −1 
x = − π/2. Но −π/2 < −π/3 — этот корень
тоже придется отбросить. А вместе с ним — и все корни
для n < −1.

Получается, что на отрезке  [−π/3; π/3] лежит только
корень x = 0,2. Отметим его вместе со знаками
и границами на координатной прямой:

Задача B11 с тригонометрической функцией

Чтобы удостовериться,
что справа от x = 0,2 производная действительно
отрицательна, достаточно подставить в y’ значение x = π/4.
Мы же просто отметим, что в точке x = 0,2
производная меняет знак с плюса на минус, а следовательно
это точка максимума.

Ответ: 0,2

Пример
19.
Найти точку максимума функции y =  (2х – 3)
cos x  – 2sin x+5

 принадлежащую
отрезку 
(0; .

Решение:  Сначала найдем производную данной функции, применив
правило для вычисления производной  произведения двух функций:

На промежутке  (0; производная обращается в нуль
только при х =1,5, поскольку
sinx>0 при  х   (0; . В точке х =1,5 производная
меняет знак с плюса на минус, эта точка является единственной точкой максимума
на данном промежутке.

Ответ: 1,5

Заключение

          При выполнении данной
работы мною были отобраны и проанализированы различные дидактические материалы,
литература, интернет ресурсы по данной теме. В результате которых, были
составлены методические рекомендации к решению заданий 12  с кратким ответом. 
Эта работа позволяет учителю подготовить учащихся по данному вопросу, а ученику
успешно справится с заданиями данной группы.

          Методика преподавания
школьного курса математики и подготовка к ЕГЭ должна быть ориентирована на
гармонизацию традиционного и инновационного подходов к обучению с применением
современных образовательных технологий, деятельностного подхода в обучении, использования
ресурсов сети Интернет.

При подготовке к ЕГЭ необходимо
использовать интернет-ресурсы:

·       
На сайте www.mioo.ru (дистанционные уроки)

·       
На сайте www.matheg.ru (открытый банк заданий I части)

А так же печатные издания:

·       
Учебники,
имеющие гриф Министерства Образования РФ;

·       
Пособия,
рекомендованные ФИПИ и МИОО для подготовки к ЕГЭ

Литература:

При разработке дидактических материалов
использованы ресурсы сети Интернет:

http://www.korolewa-ow.narod.ru/sist_ind.htm

http://www.mathnet.spb.ru/texts.htm

http://www.beluo.ru/

Материалы сайта ФИПИ (http://www.fipi.ru)

КоролеваТ.С.,
СорокинаЛ.В., Новикова Т.В., ЗеленоваЕ.В. и др.

Приложение

1.     Найдите
наибольшее значение функции y=sqrt{117+4x-x^2}.

2.     Найдите
наибольшее значение функции y=sqrt{5+4x-x^2}.

3.     Найдите
наибольшее значение функции y=sqrt{48+22x-x^2}.

4.     Найдите наибольшее
значение функции y=sqrt{-105+22x-x^2}.

5.     Найдите
наибольшее значение функции y=sqrt{-48-26x-x^2}.

6.     Найдите
наименьшее значение функции y=sqrt{x^2+12x+180}.

7.    
Найдите наименьшее значение функции y=sqrt{x^2-18x+181}

8.    
Найдите точку минимума функции y=sqrt{x^2-22x+126}.

9.    
Найдите точку минимума функции y=sqrt{x^2+22x+137}

10.  Найдите
точку максимума функции 
y=sqrt{-8+6x-x^2}.

11.  Найдите
точку максимума функции 
y=sqrt{-136+24x-x^2}

12.  Найдите
наибольшее значение функции 
y=4^{-99+20x-x^2}.

13.  Найдите
наибольшее значение функции 
y=2^{-37-12x-x^2}.

14.  Найдите
наименьшее значение функции 
y=7^{x^2-10x+28}.

15.  Найдите
наименьшее значение функции 
y=5^{x^2+16x+65}.

16.  Найдите
точку минимума функции 
y=7^{x^2-16x+78}

17.  Найдите
точку минимума функции 
y=2^{x^2+18x+97}.

18.  Найдите
точку максимума функции 
y=3^{14+8x-x^2}

19.  Найдите
точку максимума функции 
y=9^{-62-16x-x^2}.

20.  Найдите
наименьшее значение функции 
+ 19

21.  Найдите
наибольшее значение функции 
  =   –
31

22.  Найдите
точку максимума функции 
y=4^{-10x-x^2}.

23.  Найдите
наибольшее значение функции y=(x+2)^2(x-1)+1 на
отрезке [-3;0].

24.  Найдите
наибольшее значение функции 
y=(x+8)^2(x+6) на
отрезке 
[-9;-7].

25.  Найдите
наибольшее значение функции 
y=(x+10)^2x на
отрезке 
[-12;-6].

26.  Найдите
наименьшее значение функции 
y=(x+3)^2x+9 на
отрезке 
[-3;0].

27.  Найдите
точку минимума функции 
y=x^2(x-3)+9.

28.  Найдите
точку минимума функции 
y=(x-7)^2x+8

29.  Найдите
точку максимума функции 
y=x^2(x-10)-3.

30.  Найдите
точку максимума функции 
y=(x+5)^2(x+8)+5.

31.  Найдите
точку максимума функции 
y=x^2(x-1)-1

32.  Найдите наибольшее
значение функции y=x^2(x+3)-2 на отрезке [-8;-1].

33.  Найдите наибольшее
значение функции y=(x+4)^2(x-1)+5 на отрезке [-16;-1].

34.  Найдите
точку максимума функции 
y=x^3 -12x +5.

35.  Найдите
наименьшее значение функции 
y=x^3 -3x+23 на
отрезке 
[0;2].

36.  Найдите
точку максимума функции 
y=x^3 -192x +23.

37.  Найдите
наибольшее значение функции 
y=x^3 -75x+19 на
отрезке 
[-6;0].

38.  Найдите
точку минимума функции 
y=x^3 -6x^2+9x+3

39.  Найдите
наименьшее значение функции 
y=x^3 -14x^2+49x+7 на
отрезке 
[6;11].

40.  Найдите
точку минимума функции 
y=7+300x-x^3.

41.  Найдите
точку минимума функции 
y=-13,5x^2-x^3+83.

42.  Найдите
точку максимума функции 
y=frac{x^3}{3} -36x+23.

43.  Найдите
точку минимума функции 
y=frac{x^3}{3} -16x+5.

44.  Найдите
точку минимума функции 
y=x^{frac{3}{2}} -30x+2.

45.  Найдите
точку минимума функции 
y=6x^2-x^3+7.

46.  Найдите
наименьшее значение функции 
y=x^{frac{3}{2}} -6x+8 на
отрезке 
[3;403].

47.  Найдите
точку минимума функции 
y=frac{1}{3}x^{frac{3}{2}} -8x+55.

48.  Найдите точку минимума
функции y=frac{1}{3}x^{frac{3}{2}} -9x+81.

49.  Найдите
наименьшее значение функции 
y=frac{4}{3}x^{frac{3}{2}} -12x+3 на
отрезке 
[34;32].

50.  Найдите
точку максимума функции 
y=2+3x-2x^{frac{3}{2}}.

51.  Найдите
наибольшее значение функции 
y=10+6x-2x^{frac{3}{2}} на
отрезке 
[4;0].

52.  Найдите
точку максимума функции 
y=-frac{4}{3}x^{frac{3}{2}} +9x+10.

53.  Найдите
точку минимума функции 
y=xsqrt{x} -24x+29.

54.  Найдите
наименьшее значение функции 
y=xsqrt{x} -30x+18 на
отрезке 
[400;0].

55.  Найдите точку
минимума функции y=frac{2}{3}xsqrt{x} -8x+6.

56.  Найдите
точку максимума функции 
y=-frac{x^2 +900}{x}.

57.  Найдите
точку минимума функции 
y=-frac{x^2 +81}{x}

58.  Найдите
наименьшее значение функции 
y=frac{x^2 +25}{x} на
отрезке 
[1;12]

59.  Найдите
наибольшее значение функции 
y=frac{x^2 +324}{x} на
отрезке 
[-28;-1].

60.  Найдите
точку минимума функции 
y=frac{400}{x} +x+7.

61.  Найдите
наименьшее значение функции 
y=x+frac{400}{x} +7 на
отрезке 
[0,5;31].

62.  Найдите
наибольшее значение функции 
y=x+frac{64}{x} +13 на
отрезке 
[-19;-0,5].

63.  Найдите точку минимума
функции y=-frac{x}{x^2 +9}.

64.  Найдите точку минимума
функции y=-frac{x}{x^2 +121}.

65.  Найдите наименьшее
значение функции   у =
    на отрезке .

66.  Найдите наименьшее
значение функции   у =
  на отрезке .

67.  Найдите наибольшее
значение функции   у = 
.

68.  Найдите наибольшее
значение функции   у = 
.

69.  Найдите
наибольшее значение функции   у = 
.

70.  Найдите
наибольшее значение функции 
y=log_7(-42-14x-x^2)-6.

71.  Найдите
наибольшее значение функции 
y=log_7(-162+26x-x^2)-4.

72.  Найдите
наименьшее значение функции 
y=log_2(x^2+6x+41)+3.

73.  Найдите
наименьшее значение функции 
y=log_9(x^2-10x+754)+3.

74.  Найдите
точку минимума функции 
y=log_7(x^2-14x+54)+7.

75.  Найдите
точку минимума функции 
y=log_2(x^2+24x+160)+3.

76.  Найдите
точку максимума функции 
y=log_5(-19+12x-x^2)+2.

77.  Найдите
точку максимума функции 
y=log_7(-185+28x-x^2)-7.

78.  Найдите наибольшее значение
функции y=log_2(-84+20x-x^2)+10.

79.  Найдите
наименьшее значение функции 
y~=~4x-ln {{(x+8)}^{4}} на
отрезке 
[-7,5;0].

80.  Найдите
наименьшее значение функции 
y~=~7x-ln {{(x+4)}^{7}} на
отрезке 
[-3,5;0]

81.  Найдите
наибольшее значение функции 
y~=~ln {{(x+5)}^{5}}-5x на
отрезке 
[-4,5;0].

82.  Найдите
наименьшее значение функции 
y~=~4x-4ln (x+7)+6 на
отрезке 
[-6,5;0]

83.  Найдите
наименьшее значение функции 
y~=~2x-2ln (x+8)+7 на
отрезке 
[-7,5;0]

84.  Найдите
наибольшее значение функции 
y~=~ln (4x)-4x+3 на
отрезке 
[frac{1}{8};frac{5}{8}].

85.  Найдите
наибольшее значение функции 
y~=~2x^2-10x+6ln x+5 на
отрезке 
[frac{10}{11};frac{12}{11}]

86.  Найдите точку минимума
функции y=0,5x^2-14x+48ln x +8.

87.  Найдите точку минимума
функции y=1,5x^2-27x+60ln x +7.

88.  Найдите точку минимума функции y=0,5x^2-12x+32ln x -1.

89.  Найдите точку минимума
функции y=1,5x^2-39x+126ln x -8.

90.  Найдите точку минимума
функции y=x^2-26x+72ln x -7.

91.  Найдите точку минимума
функции y=x^2-24x+70ln x -3.

92.  Найдите наименьшее
значение функции    у = (х – 7)е х – 6  на отрезке

93.  Найдите наименьшее
значение функции    у = (х2  – 7х – 7)ех – 5  на отрезке

94.  Найдите наибольшее
значение функции   у =
 х2 +8.

95.  Найдите наибольшее
значение функции   у =
 х2 – 10.

96.  Найдите наибольшее
значение функции 
 + 7.

97.  Найдите наибольшее
значение функции   у =
 х2 +3.

98.  Найдите наименьшее
значение функции  
 + 1.

99.  Найдите наименьшее
значение функции  
 – 3.

100.     Найдите наибольшее значение
функции 
y=log_3(720+6x-x^2)+9.

101.     Найдите наибольшее значениефункции y=log_3(-94+22x-x^2)-4.

102.    
Найдите точку минимума функции y = 2x2-5x + lnx-3.

103.    
Найдите точку максимума функции y = (2x-3) cosx-2sinx +5 
 принадлежащую промежутку (0;π/2).

104.      Найдите наименьшее значение функции y~=~(x-8){{e}^{x-7}} на
отрезке 
[6;8]

105.     Найдите
наименьшее значение функции 
y~=~(x-23){{e}^{x-22}} на
отрезке 
[21;23].

106.                  
Найдите точку максимума функции y~=~(11-x){{e}^{x+11}}

107.                  
Найдите точку максимума функции y~=~(3-x){{e}^{x+3}}.

108.                  
Найдите точку максимума функции y~=~(21-x){{e}^{x+21}}.

109.                  
Найдите точку максимума функции y~=~(x+14){{e}^{14-x}}.

110.                  
Найдите
наибольшее значение функции 
y~=~12sqrt{2}cos x+12x-3pi +9 на       отрезке [0;frac{pi }{2}].

111.                  
Найдите наименьшее значение функции y~=~4+frac{11pi }{4}-11x-11sqrt{2}cos x на
отрезке 
[0;frac{pi }{2}].

112.                  
Найдите наименьшее значение функции y~=~13cos x-15x+7 на
отрезке 
[-frac{3pi }{2};0].

113.                  
Найдите наибольшее значение функции y~=~7x-2sin x+7 на
отрезке [-frac{pi }{2};0].

114.                  
Найдите наибольшее значение функции y~=~2sin x-frac{12}{pi }x+6 на
отрезке 
[-frac{5pi }{6};0].

115.                  
Найдите наибольшее значение функции y~=~2cos x-frac{24}{pi }x+8 на
отрезке 
[-frac{2pi }{3};0].

116.                  
Найдите наибольшее значение функции y~=~4cos x-frac{18}{pi }x+3 на
отрезке 
[-frac{2pi }{3};0]

117.                  
Найдите наименьшее значение функции y~=~3sin x+frac{30}{pi }x+3 на
отрезке 
[-frac{5pi }{6};0]

118.                  
Найдите наибольшее значение функции y~=~2tgx-2x+5 на
отрезке 
[-frac{pi }{4};0].

119.                  
Найдите наибольшее значение функции y~=~20tgx-20x+5pi -5 на
отрезке 
[-frac{pi }{4};frac{pi }{4}].

120.                  
Найдите наибольшее значение функции y~=~7x-7tgx-4 на
отрезке 
[0;frac{pi }{4}].

121.                  
Найдите наименьшее значение функции y~=~14tgx-28x+7pi -4 на
отрезке 
[-frac{pi }{3};frac{pi }{3}]

122.                  
Найдите наибольшее значение функции y~=~2x-tgx-0,5pi +13 на
отрезке 
[-frac{pi }{3};frac{pi }{3}]

123.                  
Найдите точку минимума функции y~=~(x^2-17x+17){{e}^{x-17}}.

124.                  
Найдите точку  минимума функции y~=~{{(x-2)}^{2}}{{e}^{x-5}}.

125.                  
Найдите точку минимума функции y~=~(3x^2-15x+15){{e}^{x-15}}

126.                  
Найдите точку максимума функции y~=~(2x^2-32x+32){{e}^{x+32}}

127.                  
Найдите точку максимума функции y~=~(x^2-14x+14){{e}^{x+14}}.

128.                  
Найдите точку минимума функции y~=~(3x^2-51x+51){{e}^{5-x}}

129.                  
Найдите точку максимума функции y~=~{{(x-3)}^{2}}{{e}^{x-2}}.

130.                  
Найдите точку минимума функции y = (x+34){{e}^{x-34}}.

131.                  
Найдите точку минимума функции y = (x+79){{e}^{x-79}}.

132.                  
Найдите точку максимума функции y=0,5x^2-18x+80ln x +7.

133.                  
Найдите точку максимума функции y=0,5x^2-11x+18ln x +8.

134.                  
Найдите точку минимума функции y=x^2-28x+80ln x -7

135.                  
Найдите точку минимума функции y=1,5x^2-39x+120ln x -10.

136.                  
Найдите точку максимума функции, y=(4x-6)cos x -4sin x+8 принадлежащую
промежутку 
(0;frac{pi}{2}).

137.                  
Найдите точку минимума функции, y=(3-2x)cos x +2sin x+11 принадлежащую
промежутку 
(0;frac{pi}{2}).

138.                  
Най­ди­те точку мак­си­му­ма функ­ции http://reshuege.ru/formula/aa/aaa3044d646a72cd072b8c500280a38b.png

139.                  
Най­ди­те наи­мень­шее зна­че­ние функ­ции http://reshuege.ru/formula/d2/d294a7494adeab7adc20128a4d035da7.png на
от­рез­ке http://reshuege.ru/formula/81/814ff7e5e8a815231ad4efc23b6b177b.png.

140.                  
Най­ди­те точку ми­ни­му­ма функ­ции http://reshuege.ru/formula/8e/8ea5a1f2913e0f4dec601e80bcbec635.png.

141.                  
Най­ди­те точку ми­ни­му­ма функ­ции http://reshuege.ru/formula/e6/e620e1419470bd3b5ce48b1278fa73e1.png.

142.                  
Най­ди­те наи­боль­шее зна­че­ние функ­ции http://reshuege.ru/formula/d0/d0f3084c8edea1734b5a4ce8a79ab17c.png на
от­рез­ке http://reshuege.ru/formula/c4/c48504004243ddc75358c6a0d695704b.png.

143.                  
 Най­ди­те наи­мень­шее зна­че­ние функ­ции http://reshuege.ru/formula/b7/b77066bb210dca83eea5f85662b5dac7.png на
от­рез­ке http://reshuege.ru/formula/b0/b097b20d9574309847599451f6f36976.png

144.                  
Най­ди­те наи­мень­шее зна­че­ние функ­ции http://reshuege.ru/formula/a5/a5152a0e68ab93d949d33bf65335a5a8.png на
от­рез­ке http://reshuege.ru/formula/8a/8aae8126e3731adfbfb5c24af35e3b61.png.

145.                  
Най­ди­те наи­боль­шее зна­че­ние функ­ции http://reshuege.ru/formula/02/02750a25d98fb0b1ae62f3485cd9bbd6.png на
от­рез­ке http://reshuege.ru/formula/91/91b0c8355dd669a14515d0f59057ef42.png.

Добавить комментарий